Top Banner
INSIGHTSIAS SIMPLYFYING IAS EXAM PREPARATION INSTA Tests DAYS 31 to 34 INSTA Revision Plan 3.0 - 2020 Copyright © by Insights IAS All rights are reserved. No part of this document may be reproduced, stored in a retrieval system or transmitted in any form or by any means, electronic, mechanical, photocopying, recording or otherwise, without prior permission of Insights IAS. OFFLINE Centres at BENGALURU | DELHI | HYDERABAD For more visit: www.INSIGHTSONINDIA.com SOLUTIONS
75

INSTA Revision Plan 3.0 - 2020 INSTA Tests€¦ · • State Agricultural Marketing Board made responsible for grading and standardization. 7. Consider the following statements regarding

Sep 24, 2020

Download

Documents

dariahiddleston
Welcome message from author
This document is posted to help you gain knowledge. Please leave a comment to let me know what you think about it! Share it to your friends and learn new things together.
Transcript
Page 1: INSTA Revision Plan 3.0 - 2020 INSTA Tests€¦ · • State Agricultural Marketing Board made responsible for grading and standardization. 7. Consider the following statements regarding

INSIGHTSIAS SIMPLYFYING IAS EXAM PREPARATION

INSTA Tests DAYS 31 to 34

INSTA Revision Plan 3.0 - 2020

Copyright © by Insights IAS All rights are reserved. No part of this document may be reproduced, stored in a retrieval system or transmitted in any form or by any means, electronic, mechanical, photocopying, recording or otherwise, without prior permission of Insights IAS.

OFFLINE Centres at BENGALURU | DELHI | HYDERABAD

For more visit: www.INSIGHTSONINDIA.com

SOLUTIONS

Page 2: INSTA Revision Plan 3.0 - 2020 INSTA Tests€¦ · • State Agricultural Marketing Board made responsible for grading and standardization. 7. Consider the following statements regarding
Page 3: INSTA Revision Plan 3.0 - 2020 INSTA Tests€¦ · • State Agricultural Marketing Board made responsible for grading and standardization. 7. Consider the following statements regarding

www.insightsonindia.com 1 INSTA Revision 3.0

INSIGHTSIAS SIMPLYFYING IAS EXAM PREPARATION

DAY – 31

1. Which of the following factors are taken into consideration for determination of MSP

1. International price situation

2. Effect on issue prices and implications for subsidy

3. Inter-crop price parity

Which of the statements given above is/are correct?

(a) 1 and 2 only

(b) 1 and 3 only

(c) 2 and 3 only

(d) 1, 2 and 3

Solution: D

Determination of MSP

In formulating the recommendations in respect of the level of minimum support prices and other

non-price measures, the Commission takes into account, apart from a comprehensive view of the

entire structure of the economy of a particular commodity or group of commodities, the following

factors:-

1. Cost of production

2. Changes in input prices

3. Input-output price parity

4. Trends in market prices

5. Demand and supply

6. Inter-crop price parity

7. Effect on industrial cost structure

8. Effect on cost of living

9. Effect on general price level

10. International price situation

11. Parity between prices paid and prices received by the farmers.

12. Effect on issue prices and implications for subsidy

2. Consider the following statements regarding Pricing policy for sugarcane

1. The pricing of sugarcane is governed by the statutory provisions under the Essential

Commodities Act (ECA), 1955.

Page 4: INSTA Revision Plan 3.0 - 2020 INSTA Tests€¦ · • State Agricultural Marketing Board made responsible for grading and standardization. 7. Consider the following statements regarding

www.insightsonindia.com 2 INSTA Revision 3.0

INSIGHTSIAS SIMPLYFYING IAS EXAM PREPARATION

2. States can also announce a price called the State Advisory Price (SAP) which is usually

higher than the Statutory Minimum Price.

Which of the statements given above is/are correct?

(a) 1 only

(b) 2 only

(c) Both 1 and 2

(d) Neither 1 nor 2

Solution: C

Pricing policy for sugarcane

The pricing of sugarcane is governed by the statutory provisions of the Sugarcane (Control) Order,

1966 issued under the Essential Commodities Act (ECA), 1955. Prior to 2009-10 sugar season, the

Central Government was fixing the Statutory Minimum Price (SMP) of sugarcane and farmers

were entitled to share profits of a sugar mill on 50:50 basis. As this sharing of profits remained

virtually unimplemented, the Sugarcane (Control) Order, 1966 was amended in October, 2009 and

the concept of SMP was replaced by the Fair and Remunerative Price (FRP) of sugarcane. A new

clause ‘reasonable margins for growers of sugarcane on account of risk and profits’ was inserted as

an additional factor for working out FRP and this was made effective from the 2009-10 sugar

season. Accordingly, the CACP is required to pay due regard to the statutory factors listed in the

Control Order, which are

1. the cost of production of sugarcane;

2. the return to the grower from alternative crops and the general trend of prices of

agricultural commodities;

3. the availability of sugar to the consumers at a fair price;

4. the price of sugar;

5. the recovery rate of sugar from sugarcane;

6. the realization made from sale of by-products viz. molasses, bagasse and press mud or their

imputed value (inserted in December, 2008) and;

7. reasonable margins for growers of sugarcane on account of risk and profits (inserted in

October, 2009).

States also announce a price called the State Advisory Price (SAP), which is usually higher

than the SMP.

3. Consider the following statements regarding Removal of judges

1. A judge is removable from his office, only on the grounds of proved misbehavior or

incapacity.

2. A judge may be removed from his office only by an order of the president.

3. The words “misbehaviour” and “incapacity” have been defined in the Constitution

Page 5: INSTA Revision Plan 3.0 - 2020 INSTA Tests€¦ · • State Agricultural Marketing Board made responsible for grading and standardization. 7. Consider the following statements regarding

www.insightsonindia.com 3 INSTA Revision 3.0

INSIGHTSIAS SIMPLYFYING IAS EXAM PREPARATION

Which of the pairs given above is/are correctly matched?

(a) 1 and 2 only

(b) 2 and 3 only

(c) 1 and 3 only

(d) 1, 2 and 3

Solution: A

Removal of Judges:

Article 124(4) and the Judges Inquiry Act 1968 determine the procedure of removal of the

judges:

1. A motion of impeachment addressed to the President is to be signed by at least 100

members of the Lok Sabha or 50 members of the Rajya Sabha and then delivered to

the Speaker of Lok Sabha or the Chairman of Rajya Sabha.

2. The motion is to be investigated by a Committee of 3 judges of the Supreme Court

and a distinguished jurist.

3. If the Committee finds the judge guilty of misbehavior or that he suffers from

incapacity, the motion along with the report of the committee is taken up for

consideration in the House where motion was moved.

4. The judge is then removed by the requisite majority, i.e. majority of total and 2/3 of its

members present and voting.

Key facts:

• A member of the higher judiciary, which means the Judges and Chief Justices of the

Supreme Court of India and the state High Courts, can be removed from service only

through the process of impeachment under Article 124 (4) of the Constitution.

• A judge is removable from his office, only on the grounds of proved misbehavior or

incapacity.

• A judge may be removed from his office only by an order of the president.

• The words “misbehaviour” or “incapacity” have neither been defined nor clarified in the

Constitution.

4. Consider the following statements regarding Buffer Stock Policy

1. The concept of buffer stock was first introduced during the 1st Five Year Plan.

2. Buffer stock of food grains is used for price stabilisation or market intervention to

augment supply so as to help moderate the open market prices.

Which of the statements given above is/are correct?

(a) 1 only

(b) 2 only

Page 6: INSTA Revision Plan 3.0 - 2020 INSTA Tests€¦ · • State Agricultural Marketing Board made responsible for grading and standardization. 7. Consider the following statements regarding

www.insightsonindia.com 4 INSTA Revision 3.0

INSIGHTSIAS SIMPLYFYING IAS EXAM PREPARATION

(c) Both 1 and 2

(d) Neither 1 nor 2

Solution: B

Buffer Stock Policy of the Government of India (GOI)

The concept of buffer stock was first introduced during the IVth Five Year Plan (1969-74).

Buffer stock of food grains in the Central Pool is maintained by the Government of India (GOI) /

Central Government for

1. meeting the prescribed minimum buffer stock norms for food security,

2. monthly release of food grains for supply through Targeted Public Distribution System

(TPDS) and Other Welfare Schemes (OWS),

3. meeting emergency situations arising out of unexpected crop failure, natural disasters,

etc., and

4. price stabilisation or market intervention to augment supply so as to help moderate

the open market prices.

5. Consider the following statements regarding Operation Twist

1. It has been launched by Ministry of Finance

2. It aims to ensure Fiscal consolidation

Which of the statements given above is/are correct?

(a) 1 only

(b) 2 only

(c) Both 1 and 2

(d) Neither 1 nor 2

Solution: D

Operation Twist:

• RBI launched US-style ‘Operation Twist’ to bring down interest rates.

• ‘Operation Twist’ is when the central bank uses the proceeds from sale of short-term

securities to buy long-term government debt papers, leading to easing of interest

rates on the long-term papers.

• The objective behind such an operation is management of the yield curve.

• It will help to make loans less expensive with those looking to buy homes, cars and make

savings less desirable as it doesn’t pay much interest.

Page 7: INSTA Revision Plan 3.0 - 2020 INSTA Tests€¦ · • State Agricultural Marketing Board made responsible for grading and standardization. 7. Consider the following statements regarding

www.insightsonindia.com 5 INSTA Revision 3.0

INSIGHTSIAS SIMPLYFYING IAS EXAM PREPARATION

• Other central banks, including the US Federal Reserve, have used similar measures. This is

the first time RBI has undertaken such an unconventional policy measure with the

aim of flattening the yield curve by lowering longer rates to boost lending and growth.

6. Consider the following statements regarding major features of Model APMC Act 2003

1. It provides for direct sale of farm produce to contract farming sponsors.

2. It introduced licensing for market functionaries and trade at any market area within

state.

3. It permits private persons, farmers and consumers to establish new markets for

agricultural produce in any area.

Which of the statements given above is/are correct?

(a) 1 and 2 only

(b) 1 and 3 only

(c) 2 and 3 only

(d) 1, 2 and 3

Solution: B

Model APMC Act 2003

• The monopoly of Government regulated wholesale markets has prevented development of a

competitive marketing system in the country. An efficient agricultural marketing is essential

for the development of the agriculture sector as it provides outlets and incentives for increased

production, the marketing system contribute greatly to the commercialization of subsistence

farmers. Worldwide Governments have recognized the importance of liberalized agriculture

markets. In accordance with above objectives, Model APMC act was drafted by ministry of

agriculture in 2003.

Major Features:

• It provides for direct sale of farm produce to contract farming sponsors.

• It provides a provision for setting up “Special markets” for “specified agricultural

commodities”

• It permits private persons, farmers and consumers to establish new markets for agricultural

produce in any area.

• Every market shall levy market fee on sale or purchase of agriculture commodities which

brought from within or outside state.

• Replaces licensing with registrations of market functionaries and trade at any market area

within state.

• Market Committees permitted to use its funds to create facilities like grading, standardization

and quality certification; to create infrastructure for post harvest handling of agricultural

produce and development of modern marketing system.

Page 8: INSTA Revision Plan 3.0 - 2020 INSTA Tests€¦ · • State Agricultural Marketing Board made responsible for grading and standardization. 7. Consider the following statements regarding

www.insightsonindia.com 6 INSTA Revision 3.0

INSIGHTSIAS SIMPLYFYING IAS EXAM PREPARATION

• State Governments conferred power to exempt any agricultural produce brought for sale in

market area, from payment of market fee.

• State Agricultural Marketing Board made responsible for grading and standardization.

7. Consider the following statements regarding E-NAM (National Agriculture Market)

1. It is an online trading platform for agriculture produce aiming to help farmers to trade

online and get a better price.

2. NABARD is the lead agency for implementing eNAM.

Which of the statements given above is/are correct?

(a) 1 only

(b) 2 only

(c) Both 1 and 2

(d) Neither 1 nor 2

Solution: A

What is e-NAM?

• E-NAM (National Agriculture Market) is an online trading platform for agriculture

produce aiming to help farmers, traders, and buyers with online trading and getting a better

price by smooth marketing.

• It was launched by the Centre in 2015 and the government had to extend it in a phased

manner across the 585 mandis of the country by December 31, 2019.

• Small Farmers Agribusiness Consortium (SFAC) is the lead agency for implementing

eNAM under the aegis of Ministry of Agriculture and Farmers’ Welfare, Government of

India.

NAM has the following advantages:

• For the farmers, NAM promises more options for sale. It would increase his access to markets

through warehouse based sales and thus obviate the need to transport his produce to the

mandi.

• For the local trader in the mandi / market, NAM offers the opportunity to access a larger

national market for secondary trading.

• Bulk buyers, processors, exporters etc. benefit from being able to participate directly in trading

at the local mandi / market level through the NAM platform, thereby reducing their

intermediation costs.

• The gradual integration of all the major mandis in the States into NAM will ensure common

procedures for issue of licences, levy of fee and movement of produce.

• The NAM will also facilitate the emergence of value chains in major agricultural commodities

across the country and help to promote scientific storage and movement of agri goods.

Page 9: INSTA Revision Plan 3.0 - 2020 INSTA Tests€¦ · • State Agricultural Marketing Board made responsible for grading and standardization. 7. Consider the following statements regarding

www.insightsonindia.com 7 INSTA Revision 3.0

INSIGHTSIAS SIMPLYFYING IAS EXAM PREPARATION

8. Which of the followings are considered as Alternative Investment Funds?

1. Venture Capital Fund

2. Hedge funds

3. Commodity funds

4. Debt Funds

5. Infrastructure funds

Select the correct answer using the code given below:

(a) 1, 2 and 3 only

(b) 1, 2 and 4 only

(c) 1, 2 and 5 only

(d) 1, 2, 3, 4 and 5

Solution: D

Alternative Investment Funds are a class of investment entities that are not covered under

the usual SEBI regulatory framework for investment institutions. AIFs refers to any privately

pooled investment fund – a trust or a company or a body corporate or an LLP (Limited Liability

Partnership) which are not presently covered by any Regulation of RBI, SEBI, IRDA and PFRDA.

They may be foreign or Indian.

• A notable general feature of AIFs is that they are tailor made investment arrangements like

Private Equities that aims to utilize investment opportunities. AIFs are thus private

investment entities.

• Thus, AIFs includes Private Equities, Venture Capital Fund, Hedge funds, Commodity funds,

Debt Funds, infrastructure funds, etc. Most of these investment entities are owned by big

corporate houses or wealthy individuals. Private Equities like Blackstone and KKR (Kohlberg

Kravis Roberts) are examples for AIFs. Several multinational banks have also AIFs. Venture

Capital Funds and Angel Investors are also categorized as AIFs.

9. Consider the following statements regarding Agricultural and Processed Food Products

Export Development Authority (APEDA)

1. APEDA was established as a statutory body under the Agricultural and Processed Food

Products Export Development Authority Act.

2. It is under Ministry of Commerce and Industry.

3. The secretary to Ministry of Commerce and Industry is the ex-officio Chairman of

APEDA.

Which of the statements given above is/are correct?

(a) 1 only

(b) 1 and 2 only

Page 10: INSTA Revision Plan 3.0 - 2020 INSTA Tests€¦ · • State Agricultural Marketing Board made responsible for grading and standardization. 7. Consider the following statements regarding

www.insightsonindia.com 8 INSTA Revision 3.0

INSIGHTSIAS SIMPLYFYING IAS EXAM PREPARATION

(c) 2 and 3 only

(d) 1, 2 and 3

Solution: A

The Agricultural and Processed Food Products Export Development Authority (APEDA)

was established by the Government of India under the Agricultural and Processed Food

Products Export Development Authority Act passed by the Parliament in December, 1985.

The Act (2 of 1986) came into effect from 13th February, 1986 by a notification issued in the Gazette

of India: Extraordinary: Part-II [Sec. 3(ii): 13.2.1986). The Authority replaced the Processed Food

Export Promotion Council (PFEPC).

It is under Ministry of Commerce and Industry.

As prescribed by the statute, the APEDA Authority consists of the following members namely:

• A Chairman, appointed by the Central Government

• The Agricultural Marketing Advisor to the Government of India, ex-offical.

• One member appointed by the Central Government representing the Planning Commission

• Three members of Parliament of whom two are elected by the House of People and one by the

Council of States

• Eight members appointed by the Central Government representing respectively; the

Ministries of the Central Govt.

10. Paramarsh Scheme has been launched by

(a) Ministry of Micro, Small and Medium Enterprises

(b) Ministry of Health

(c) Ministry of Human Resource Development

(d) Ministry of Skill Development and Entrepreneurship

Solution: C

Paramarsh Scheme

The Union Ministry of Human Resource Development has launched ‘Paramarsh’ – a

University Grants Commission (UGC)scheme.

The scheme is for Mentoring National Accreditation and Assessment Council (NAAC)

Accreditation Aspirant Institutions to promote Quality Assurance in Higher Education.

The scheme will be a paradigm shift in the concept of mentoring of institution by another well

performing institution to upgrade their academic performance and enable them to get accredited

by focusing in the area of curricular aspects, teaching-learning & evaluation, research, innovation,

institutional values & practices etc.

Page 11: INSTA Revision Plan 3.0 - 2020 INSTA Tests€¦ · • State Agricultural Marketing Board made responsible for grading and standardization. 7. Consider the following statements regarding

www.insightsonindia.com 9 INSTA Revision 3.0

INSIGHTSIAS SIMPLYFYING IAS EXAM PREPARATION

11. Which of the following reports are published by FAO

1. The State of Food and Agriculture

2. The State of the World’s Forests

3. The State of World Fisheries and Aquaculture

Which of the statements given above is/are correct?

(a) 1 and 2 only

(b) 1 and 3 only

(c) 2 and 3 only

(d) 1, 2 and 3

Solution: D

Flagship publications

Every year, FAO publishes a number of major ‘State of the World’ reports related to food,

agriculture, forestry, fisheries and natural resources.

• The State of Agricultural Commodity Markets

• The State of Food and Agriculture

• The State of Food Security and Nutrition in the World

• The State of the World’s Animal Genetic Resources for Food and Agriculture

• The State of the World’s Biodiversity for Food and Agriculture

• The State of the World’s Forest Genetic Resources

• The State of the World’s Forests

• The State of the World’s Land and Water Resources for Food and Agriculture

• The State of the World’s Plant Genetic Resources for Food and Agriculture

• The State of World Fisheries and Aquaculture

• Status of the World’s Soil Resources

12. Which of the following sites have been designated as Globally Important Agricultural

Heritage Systems (GIAHS)?

1. Saffron Heritage of Kashmir

2. Koraput Traditional Agriculture

3. Kuttanad below Sea Level Farming System

Which of the statements given above is/are correct?

(a) 1 and 2 only

Page 12: INSTA Revision Plan 3.0 - 2020 INSTA Tests€¦ · • State Agricultural Marketing Board made responsible for grading and standardization. 7. Consider the following statements regarding

www.insightsonindia.com 10 INSTA Revision 3.0

INSIGHTSIAS SIMPLYFYING IAS EXAM PREPARATION

(b) 1 and 3 only

(c) 2 and 3 only

(d) 1, 2 and 3

Solution: D

The concept of Globally Important Agricultural Heritage Systems (GIAHS) is distinct from,

and more complex than, a conventional heritage site or protected area/landscape. A GIAHS is a

living, evolving system of human communities in an intricate relationship with their territory,

cultural or agricultural landscape or biophysical and wider social environment.

Designated sites mean the sites which have already been designated as GIAHS throughout

designation process.

1. Saffron Heritage of Kashmir 2011

2. Koraput Traditional Agriculture 2012

3. Kuttanad Below Sea Level Farming System 2013

13. Consider the following statements regarding the Solar Energy Corporation of India (SECI):

1. It is under the administrative control of the Ministry of Power.

2. It was originally registered under Section 25 of Companies Act, 1956, as a Company not

for profit.

Which of the statements given above is/are correct?

(a) 1 only

(b) 2 only

(c) Both 1 and 2

(d) Neither 1 nor 2

Solution: A

• “Solar Energy Corporation of India ltd” (SECI) is a CPSU under the administrative

control of the Ministry of New and Renewable Energy (MNRE), set up on 20th Sept, 2011

to facilitate the implementation of JNNSM and achievement of targets set therein.

• It was originally incorporated as a section-25 (not for profit) company under the Companies

Act, 1956.

• However, through a Government of India decision, the company has recently been converted

into a Section-3 company under the Companies Act, 2013. The mandate of the company

has also been broadened to cover the entire renewable energy domain.

Page 13: INSTA Revision Plan 3.0 - 2020 INSTA Tests€¦ · • State Agricultural Marketing Board made responsible for grading and standardization. 7. Consider the following statements regarding

www.insightsonindia.com 11 INSTA Revision 3.0

INSIGHTSIAS SIMPLYFYING IAS EXAM PREPARATION

14. Which of the following statements compound the effect of desertification?

1. Radioactive substances from nuclear plants which are released into the soil.

2. An increase in the use of fertilizers for agriculture.

3. Open defecation by animals and human beings.

Which of the statements given above is/are correct?

(a) 1 and 2 only

(b) 1 and 3 only

(c) 2 and 3 only

(d) 1, 2 and 3

Solution: D

Soil Pollution

• Land is a very valuable but limited resource, as the population increases rapidly. Many highly

urbanised cities are faced with acute space problems, as in calcutta or bombay. Besides the

limited availability of land, 175 million hectares of land are becoming less productive every

year. India loses 20 tons of topsoil per hectare in a year due to floods, rainfall and

deforestation. 20 % to 50 % of lands under irrigation can go out of cultivation at this rate

because of water logging and salinity.

The scenario of desertification is compounded by pollution which includes

1. Indiscriminate discharge of industrial effluents on land and into water bodies

2. An increase in the use of fertilisers for agriculture

3. Open defecation by animals and human beings

4. Accumulation of solid waste; this is a major problem in developed countries like india

where the garbage and refuse products are not degraded

5. Radioactive substances from nuclear plants which are released into the soil

15. Which of the following Wetlands in India have been included in the Montreux Record?

1. Loktak Lake

2. Chilka Lake

3. Keoladeo National Park

4. Sambhar Lake

Select the correct answer using the code given below.

(a) 1 only

(b) 2 only

Page 14: INSTA Revision Plan 3.0 - 2020 INSTA Tests€¦ · • State Agricultural Marketing Board made responsible for grading and standardization. 7. Consider the following statements regarding

www.insightsonindia.com 12 INSTA Revision 3.0

INSIGHTSIAS SIMPLYFYING IAS EXAM PREPARATION

(c) 1 and 3 only

(d) 3 and 4 only

Solution: C

• Wetlands are areas of fen, marsh, peatland, or water, whether artificial or natural, temporary

or permanent, with water that is flowing or static, brackish, fresh or salt, including marine

water areas, the depth of which at low tide does not exceed six meters.

• The Montreux Record should be employed to identify priority sites for positive international

and national attention towards conservation. Sites may be removed from and added to the

Record only with Contracting Parties’ approval in which they lie.

• Chilka and Sambar lake are not included in Montreux record.

16. Consider the following statements regarding Soil Health Card (SHC)

1. It is being implemented through the Department of Agriculture of all the State and

Union Territory Governments.

2. The one of the objectives of the Soil Health Card (SHC) scheme is to issue soil health

cards to farmers every five years.

Which of the statements given above is/are correct?

(a) 1 only

(b) 2 only

(c) Both 1 and 2

(d) Neither 1 nor 2

Solution: A

• Soil Health Card (SHC) is a Government of India’s scheme promoted by the Department

of Agriculture & Co-operation under the Ministry of Agriculture and Farmers’

Welfare. It is being implemented through the Department of Agriculture of all the State

and Union Territory Governments.

• The objectives of the Soil Health Card (SHC) scheme is to issue soil health cards to farmers

every two years so as to provide a basis to address nutritional deficiencies in fertilization

practices. Soil testing is developed to promote soil test based on nutrient management. Soil

testing reduces cultivation cost by application of right quantity of fertilizer. It ensures

additional income to farmers by increase in yields and it also promotes sustainable farming.

• The scheme has been introduced to assist State Governments to issue SHCs to all farmers in

the country. SHC provides information to farmers on nutrient status of their soil along with

recommendation on appropriate dosage of nutrients to be applied for improving soil health

and its fertility.

Page 15: INSTA Revision Plan 3.0 - 2020 INSTA Tests€¦ · • State Agricultural Marketing Board made responsible for grading and standardization. 7. Consider the following statements regarding

www.insightsonindia.com 13 INSTA Revision 3.0

INSIGHTSIAS SIMPLYFYING IAS EXAM PREPARATION

• Deterioration of soil chemical, physical and biological health is considered as one of the

reasons for stagnation of agricultural productivity in India.

17. Consider the following statements regarding the Aflatoxins:

1. They are family of toxins produced by certain bacteria.

2. Exposure to aflatoxins is associated with an increased risk of liver cancer.

Which of the statements given above is/are correct?

(a) 1 only

(b) 2 only

(c) Both 1 and 2

(d) Neither 1 nor 2

Solution: B

Aflatoxins are a family of toxins produced by certain fungi that are found on agricultural crops

such as maize (corn), peanuts, cottonseed, and tree nuts.

• The main fungi that produce aflatoxins are Aspergillus flavus and Aspergillus

parasiticus, which are abundant in warm and humid regions of the world. Aflatoxin-

producing fungi can contaminate crops in the field, at harvest, and during storage.

• Exposure to aflatoxins is associated with an increased risk of liver cancer..

18. Consider the following statements regarding Bio-gas

1. It is a combination of methane and carbon dioxide.

2. It is formed by anaerobic digestion of organic materials.

3. It normally burns without smoke and has a high heating capacity.

Which of the statements given above is/are correct?

(a) 1 and 2 only

(b) 1 and 3 only

(c) 2 and 3 only

(d) 1, 2 and 3

Solution: D

• Biogas typically refers to a gas produced by the breakdown of organic matter in the absence

of oxygen. It is a renewable energy source, like solar and wind energy. Furthermore, biogas

Page 16: INSTA Revision Plan 3.0 - 2020 INSTA Tests€¦ · • State Agricultural Marketing Board made responsible for grading and standardization. 7. Consider the following statements regarding

www.insightsonindia.com 14 INSTA Revision 3.0

INSIGHTSIAS SIMPLYFYING IAS EXAM PREPARATION

can be produced from regionally available raw materials, recycled waste, and is

environmentally friendly.

• Biogas is produced by the anaerobic digestion or fermentation of biodegradable materials such

as manure, sewage, municipal waste, green waste, plant material, and crops.

• Biogas comprises primarily methane (CH4) and carbon dioxide (CO2) and may have

small amounts of hydrogen sulphide (H2S), moisture and siloxanes. Biogas is produced

by the anaerobic digestion or fermentation of biodegradable materials such as manure,

sewage, municipal waste, green waste, plant material, and crops.

• http://www.nbrienvis.nic.in/Database/1_2042.aspx

19. Bhashan Char Island, sometimes seen in news, belongs to:

(a) Myanmar

(b) India

(c) Bangladesh

(d) Sri Lanka

Solution: C

Bhasan Char, also known as Char Piya, is an island in Sandwip Upazila, Bangladesh. It is

located in the Bay of Bengal, 37 miles from the coast. The island was formed by Himalayan silt in

2006.

It was in news because of rehabilitation of Rohingyas.

20. Which of the following animals can be declared as vermin?

1. Golden Langur

2. Rhesus Monkey

3. Nilgai

4. Indian Bustard

Select the correct answer using the code given below:

(a) 1 and 2 only

(b) 2 and 3 only

(c) 2, 3 and 4 only

(d) 1 and 3 only

Solution: B

Page 17: INSTA Revision Plan 3.0 - 2020 INSTA Tests€¦ · • State Agricultural Marketing Board made responsible for grading and standardization. 7. Consider the following statements regarding

www.insightsonindia.com 15 INSTA Revision 3.0

INSIGHTSIAS SIMPLYFYING IAS EXAM PREPARATION

• Uttarakhand, Bihar and Himachal Pradesh have had notifications issued by the Central

Government stating that select species of wildlife have been declared as vermin in specified

areas.

• In other words for a specified period, in these areas, the wild pig in Uttarakhand, wild pig

and Nilgai in Bihar and Rhesus Macaque in Himachal Pradesh are shifted to schedule

five of the Wildlife Protection Act. Besides, Maharashtra and Telangana have had culling

orders issued by the Chief Wildlife Wardens.

• The latter has even appointed a panel of hunters! These decisions – for species that are

damaging human life or property (including crops) – appear to be ad hoc and random.

• Asian Elephant and Indian Bustard are included in Schedule 1 of WPA, 1972.

https://www.conservationindia.org/articles/legalising-hunting

21. Mitra Shakti, a defence exercise, conducted between India and

(a) UAE

(b) Sri Lanka

(c) France

(d) Oman

Solution: B

The seventh edition of the joint training exercise between the Indian Army and the Sri

Lankan Army, Exercise Mitra Shakti concluded on 14 December, 2019 at Aundh Military

Station, Pune. Contingent comprising of 120 personnel each from Sri Lankan and Indian Army

participated in the exercise.

22. Consider the following statements:

1. The National Bank for Agriculture and Rural Development (NABARD) was established

on the recommendation of M.S. Swaminathan committee

2. The twenty-point programme was launched during the tenth five year plan.

Which of the statements given above is/are correct?

(a) 1 only

(b) 2 only

(c) Both 1 and 2

(d) Neither 1 nor 2

Solution: D

Page 18: INSTA Revision Plan 3.0 - 2020 INSTA Tests€¦ · • State Agricultural Marketing Board made responsible for grading and standardization. 7. Consider the following statements regarding

www.insightsonindia.com 16 INSTA Revision 3.0

INSIGHTSIAS SIMPLYFYING IAS EXAM PREPARATION

National Bank for Agriculture and Rural Development (NABARD) is an apex development

financial institution in India. It is an institution fully owned by Government of India. NABARD

was established on the recommendations of B. Sivaraman Committee, (by Act 61, 1981 of

Parliament) in 1982 to implement the National Bank for Agriculture and Rural

Development Act 1981. It replaced the Agricultural Credit Department (ACD) and Rural

Planning and Credit Cell (RPCC) of Reserve Bank of India, and Agricultural Refinance and

Development Corporation (ARDC).

The twenty-point programme was launched during the Fifth five year plan in 1975 by the

then Prime Minister Indira Gandhi. The basic objective of the 20-Point Programme is to

eradicate poverty and to improve the quality of life of the poor and the under privileged

population of the country.

23. With reference to ‘Asia Africa Growth Corridor’, consider the following statements

1. It is an initiative led by India and China to enhance connectivity between Asia and

Africa.

2. The corridor will focus on areas such as Development Cooperation Projects, Quality

Infrastructure and Institutional Connectivity.

Which of the statement(s) given above is/are correct?

(a) 1 only

(b) 2 only

(c) Both 1 and 2

(d) Neither 1 nor 2

Solution: B

Asia Africa Growth Corridor (AAGC), a fresh initiative led by India and Japan to enhance

connectivity between the two continents, was unveiled by Prime Minister of India, Narendra Modi

during the African Development Bank annual meeting in Gandhi Nagar, Gujarat on May 16, 2017.

It proposes four major pillars to bring peoples, goods, services, capital and institutions closer

together and help realise the objectives of the Asia–Africa partnership for sustainable and

innovative development.

1. development and cooperation projects;

2. quality infrastructure and institutional connectivity;

3. enhancing capacities and skills; and

4. people-to-people partnership.

24. The International Land Coalition (ILC) has been hosted by:

(a) The Food and Agriculture Organization (FAO)

(b) The International Fund for Agricultural Development (IFAD)

Page 19: INSTA Revision Plan 3.0 - 2020 INSTA Tests€¦ · • State Agricultural Marketing Board made responsible for grading and standardization. 7. Consider the following statements regarding

www.insightsonindia.com 17 INSTA Revision 3.0

INSIGHTSIAS SIMPLYFYING IAS EXAM PREPARATION

(c) World Bank

(d) United Nations Environment Program (UNEP)

Solution: B

The International Land Coalition (ILC) is a global alliance of over 200 civil society and

intergovernmental organizations in 64 countries.

• The ILC has been hosted by IFAD since it was founded in 1995. It continues to emphasize that

access to land and natural resources is fundamental to IFAD’s efforts to reduce poverty,

increase food and nutrition security, and strengthen resilience in rural areas.

• ILC’s goal is land governance that recognizes people’s dignity and human rights. The aim is

to place the people who live on and from the land at the center of decision-making, including

about their food systems. This includes women, men, young people, and indigenous

communities of both current and future generations.

• Through the Coalition, they work together to put people at the center of land governance.

25. Consider the following statements

1. International Intellectual Property (IP) Index is released by the US Chambers of

Commerce.

2. United States and China remained top two economies on the index.

Which of the statements given above is/are correct?

(a) 1 only

(b) 2 only

(c) Both 1 and 2

(d) Neither 1 nor 2

Solution: A

International Intellectual Property Index

• India has slipped to 40th position on the International Intellectual Property (IP) Index,

2020 from the 36th position in 2019.

• It is released by the US Chamber of Commerce’s Global Innovation Policy Center. The

US, the UK, France, Germany and Sweden are the top five economies on the IP Index in 2020.

Page 20: INSTA Revision Plan 3.0 - 2020 INSTA Tests€¦ · • State Agricultural Marketing Board made responsible for grading and standardization. 7. Consider the following statements regarding

www.insightsonindia.com 18 INSTA Revision 3.0

INSIGHTSIAS SIMPLYFYING IAS EXAM PREPARATION

DAY – 32

26. Consider the following statements regarding GDP

1. Agriculture, forestry and fishing constitute around 12 percent of GDP.

2. Mining, quarrying, electricity, gas and water supply constitute around 15 percent of

GDP.

Which of the statements given above is/are correct?

(a) 1 only

(b) 2 only

(c) Both 1 and 2

(d) Neither 1 nor 2

Solution: A

The most important and the fastest growing sector of Indian economy are services. Trade, hotels,

transport and communication; financing, insurance, real estate and business services and

community, social and personal services account for more than 60 percent of GDP. Agriculture,

forestry and fishing constitute around 12 percent of the output, but employs more than 50 percent

of the labor force. Manufacturing accounts for 15 percent of GDP, construction for another 8 percent

and mining, quarrying, electricity, gas and water supply for the remaining 5 percent.

27. Consider the following statements regarding Disinvestment policy in India

1. Disinvestment policy is implemented by Department for revenue, Ministry of Finance.

2. For the last 5 financial years, total receipts from the disinvestment has been increasing

over consecutive years.

Which of the statements given above is/are correct?

(a) 1 only

(b) 2 only

(c) Both 1 and 2

(d) Neither 1 nor 2

Solution: D

• The renaming and restructuring of the Department of Disinvestment were announced by the

Finance Minister in his 2016-17 budget speech.

Page 21: INSTA Revision Plan 3.0 - 2020 INSTA Tests€¦ · • State Agricultural Marketing Board made responsible for grading and standardization. 7. Consider the following statements regarding

www.insightsonindia.com 19 INSTA Revision 3.0

INSIGHTSIAS SIMPLYFYING IAS EXAM PREPARATION

• As a follow-up, the Dept of Disinvestment has been renamed as the Department of

Investment and Public Asset Management or ‘DIPAM’ but it continues to function under

the Ministry of Finance.

• Aim of DIPAM: Efficient management of center’s investments in equity including its

disinvestment in central public sector undertakings (CPSU).

28. Consider the following statements regarding Cabinet Secretary

1. The cabinet secretariat is under the direct charge of the prime minister.

2. Cabinet secretary is appointed for a fixed tenure of five years.

Which of the statements given above is/are correct?

(a) 1 only

(b) 2 only

(c) Both 1 and 2

(d) Neither 1 nor 2

Solution: A

Cabinet Secretary:

• A cabinet secretary is appointed for a fixed tenure of two years.

• According to All India Services (Death-Cum-Retirement-Benefits) Rules, 1958, the

government can give extension in service to a cabinet secretary provided the total tenure

does not exceed four years.

Page 22: INSTA Revision Plan 3.0 - 2020 INSTA Tests€¦ · • State Agricultural Marketing Board made responsible for grading and standardization. 7. Consider the following statements regarding

www.insightsonindia.com 20 INSTA Revision 3.0

INSIGHTSIAS SIMPLYFYING IAS EXAM PREPARATION

• As per the modified rules, the central government may give an extension in service for a

further period not exceeding three months, beyond the period of four years to a cabinet

secretary.

Role of the cabinet secretary:

• The cabinet secretariat is under the direct charge of the prime minister.

• The administrative head of the secretariat is the cabinet secretary who is also the ex-

officio chairman of the civil services board.

Functions:

• The cabinet secretariat assists in decision-making in government by ensuring inter-

ministerial coordination, ironing out differences amongst ministries or departments and

evolving consensus through the instrumentality of the standing or ad hoc committees of

secretaries.

• Management of major crisis situations in the country and coordinating activities of various

ministries in such a situation is also one of the functions of the cabinet secretariat.

• Cabinet Secretariat is responsible for the administration of the Government of India

(Transaction of Business) Rules, 1961 and the Government of India (Allocation of

Business) Rules 1961, facilitating smooth transaction of business in Ministries/

Departments of the Government.

29. Consider the following statements regarding different approaches to disinvestments

1. Minority

disinvestment

: The government retains a majority stake (typically more than

51%) in the company and it ensures management control

2. Strategic

Disinvestment

: It is a form of majority disinvestment wherein 100% control of

the company is passed on to a buyer

Which of the statements given above is/are correct?

(a) 1 only

(b) 2 only

(c) Both 1 and 2

(d) Neither 1 nor 2

Solution: A

There are primarily three different approaches to disinvestments:

1. Minority disinvestment: The government retains a majority stake (typically more than

51%) in the company and it ensures management control. Some examples of minority

disinvestment via Offer for Sale include recent issues of Power Grid Corp. of India Ltd.,

Rural Electrification Corp. Ltd., NTPC Ltd, NHPC Ltd, etc.

2. Majority disinvestment: The government retains a minority stake in the company i.e. it

sells off a majority stake. It is also called Strategic Disinvestment. These strategic partners

could be other Central Public Sector Enterprises (CPSEs) themselves, a few examples

being sale of BRPL/MRL to Indian Oil Corporation Ltd. (IOC) and KRL to BPCL.

Page 23: INSTA Revision Plan 3.0 - 2020 INSTA Tests€¦ · • State Agricultural Marketing Board made responsible for grading and standardization. 7. Consider the following statements regarding

www.insightsonindia.com 21 INSTA Revision 3.0

INSIGHTSIAS SIMPLYFYING IAS EXAM PREPARATION

Alternatively, these strategic partners can be private entities, like the sale of Modern Foods

to Hindustan Lever Ltd., CMC to Tata Consultancy Services Ltd. (TCS).

3. Complete disinvestment or privatization: It is a form of majority disinvestment

wherein 100% control of the company is passed on to a buyer i.e. government completely

disinvests from that PSU. Example of this includes 18 hotel properties of India Tourism

Development Corporation (ITDC).

30. ‘Zo Kutpui’ festival is celebrated in

(a) Assam

(b) Manipur

(c) Mizoram

(d) Meghalaya

Solution: C

‘Zo Kutpui’ festival

• Mizoram govt organised ‘Zo Kutpui’ globally to unify Mizo tribes.

• The first festival would be held at Vanghmun, a hub town of Mizos in neighbouring Tripura.

• The event will see the presence of many important dignitaries from various Mizoram tribes

and also witness cultural programmes and traditional songs from various Mizo tribes from

Mizoram and other northeastern states.

• The event aims at re-unifying and strengthening brotherhood among different tribes of Mizo.

31. Consider the following statements regarding Confederation of Indian Industry (CII)

1. CII is a non-government, not-for-profit, industry-led and industry-managed

organization.

2. It has members from both the private and public sectors.

Which of the statements given above is/are correct?

(a) 1 only

(b) 2 only

(c) Both 1 and 2

(d) Neither 1 nor 2

Solution: C

Page 24: INSTA Revision Plan 3.0 - 2020 INSTA Tests€¦ · • State Agricultural Marketing Board made responsible for grading and standardization. 7. Consider the following statements regarding

www.insightsonindia.com 22 INSTA Revision 3.0

INSIGHTSIAS SIMPLYFYING IAS EXAM PREPARATION

• The Confederation of Indian Industry (CII) works to create and sustain an environment

conducive to the development of India, partnering industry, Government and civil society,

through advisory and consultative processes.

• For 125 years, CII has been working on shaping India’s development journey and, this year,

more than ever before, it will continue to proactively transform Indian industry’s engagement

in national development.

• CII is a non-government, not-for-profit, industry-led and industry-managed organization, with

about 9100 members from the private as well as public sectors, including SMEs and MNCs,

and an indirect membership of over 300,000 enterprises from 288 national and regional

sectoral industry bodies.

32. Consider the following statements regarding National Policy on Electronics (NPE)

1. It creates Sovereign Patent Fund (SPF) to promote the development and acquisition of

IPs in ESDM sector.

2. The policy envisaged the creation of US$ 40 billion electronics manufacturing industry

in the country by 2025.

3. It promote trusted electronics value chain initiatives to improve national cyber security

profile.

Which of the statements given above is/are correct?

(a) 1 and 2 only

(b) 1 and 3 only

(c) 2 and 3 only

(d) 1, 2 and 3

Solution: B

• National Policy on Electronics (NPE), passed by the Union Cabinet in February 2019,

envisaged the creation of Rs 27.96 lakh crore (US$ 400 billion) electronics manufacturing

industry in the country by 2025. 32 per cent growth rate has been targeted globally over the

next five years. Electronics manufacturing is expected to increase at an annual rate of 30 per

cent over the next five years and clock Rs 11.5 lakh crore (US$ 163.14 billion) additional

production during this period.

• The Union Cabinet gave its approval to the National Policy on Electronics 2019 (NPE 2019),

proposed by the Ministry of Electronics and Information Technology (MeitY). The Policy

envisions positioning India as a global hub for Electronics System Design and

Manufacturing – (ESDM) by encouraging and driving capabilities in the country for

developing core components, including chipsets, and creating an enabling environment for the

industry to compete globally.

Salient Features of NPE 2019

• Create eco-system for globally competitive ESDM sector: Promoting domestic manufacturing

and export in the entire value-chain of ESDM.

Page 25: INSTA Revision Plan 3.0 - 2020 INSTA Tests€¦ · • State Agricultural Marketing Board made responsible for grading and standardization. 7. Consider the following statements regarding

www.insightsonindia.com 23 INSTA Revision 3.0

INSIGHTSIAS SIMPLYFYING IAS EXAM PREPARATION

• Provide incentives and support for manufacturing of core electronic components.

• Provide special package of incentives for mega projects which are extremely high-tech and

entail huge investments, such as semiconductor facilities display fabrication, etc.

• Formulate suitable schemes and incentive mechanisms to encourage new units and expansion

of existing units.

• Promote Industry-led R&D and innovation in all sub-sectors of electronics, including grass

root level innovations and early stage Start-ups in emerging technology areas such as 5G, loT/

Sensors, Artificial Intelligence (Al), Machine Learning, Virtual Reality (VR), Drones, Robotics,

Additive Manufacturing, Photonics, Nano-based devices, etc.

• Provide incentives and support for significantly enhancing availability of skilled

manpower, including re-skilling.

• Special thrust on Fabless Chip Design Industry, Medical Electronic Devices Industry,

Automotive Electronics Industry and Power Electronics for Mobility and Strategic Electronics

Industry.

• Create Sovereign Patent Fund (SPF) to promote the development and acquisition of IPs in

ESDM sector.

• Promote trusted electronics value chain initiatives to improve national cyber security profile.

33. Consider the following statements regarding Hampi

1. It is located near the Malaprabha River

2. It is a UNESCO world heritage site

3. It was a part of the Mauryan Empire back in the third century BC

Which of the statements given above is/are correct?

(a) 1 and 3 only

(b) 2 and 3 only

(c) 2 only

(d) 1, 2 and 3

Solution: B

Hampi:

• It is a UNESCO world heritage site.

• It was a part of the Mauryan Empire back in the third century BC.

• Hampi was the capital city during the four different dynasties altogether in the Vijayanagar

city that came into existence in the year 1336 AD.

• The Vijayanagara Empire reached unfathomable heights under the guidance of King

Krishnadeva Raya of the Tuluva Dynasty.

Page 26: INSTA Revision Plan 3.0 - 2020 INSTA Tests€¦ · • State Agricultural Marketing Board made responsible for grading and standardization. 7. Consider the following statements regarding

www.insightsonindia.com 24 INSTA Revision 3.0

INSIGHTSIAS SIMPLYFYING IAS EXAM PREPARATION

• ‘Kishkindha Kaand’ in Ramayana has special significance concerning Hampi.

• It is located near the Tungabhadra river.

• By 1500 CE, Hampi-Vijayanagara was the world’s second-largest medieval-era city

after Beijing, and probably India’s richest at that time, attracting traders from Persia and

Portugal.

• It has been described by UNESCO as an “austere, grandiose site” of more than 1,600

surviving remains of the last great Hindu kingdom in South India.

34. Consider the following statements regarding India Brand Equity Foundation (IBEF)

1. It is a statutory body established under the Ministry of Commerce and Industry.

2. IBEF’s primary objective is to promote and create international awareness of the Made

in India label in markets overseas.

Which of the statements given above is/are correct?

(a) 1 only

(b) 2 only

(c) Both 1 and 2

(d) Neither 1 nor 2

Solution: B

India Brand Equity Foundation (IBEF) is a Trust established by the Department of

Commerce, Ministry of Commerce and Industry, Government of India. IBEF’s primary

objective is to promote and create international awareness of the Made in India label in markets

overseas and to facilitate dissemination of knowledge of Indian products and services. Towards

this objective, IBEF works closely with stakeholders across government and industry

India, today, is well established as a credible business partner, preferred investment destination,

rapidly growing market, provider of quality services and manufactured products; and, stands on

the threshold years of unprecedented growth.

India’s Talent, Markets, Growth and Opportunity drive Brand India.

http://www.ibef.org is a knowledge centre for global investors, international policy-makers and

world media seeking updated, accurate and comprehensive information on the Indian economy,

states and sectors. IBEF regularly tracks government announcements in policy, foreign

investment, macroeconomic indicators and business trends.

IBEF works with a network of stakeholders – domestic and international – to promote Brand India.

35. Consider the following statements regarding Eat Right Movement:

1. It was launched by NITI Aayog

2. It aims to cut down salt/sugar and oil consumption by 30% in three years.

Page 27: INSTA Revision Plan 3.0 - 2020 INSTA Tests€¦ · • State Agricultural Marketing Board made responsible for grading and standardization. 7. Consider the following statements regarding

www.insightsonindia.com 25 INSTA Revision 3.0

INSIGHTSIAS SIMPLYFYING IAS EXAM PREPARATION

Which of the statements given above is/are correct?

(a) 1 only

(b) 2 only

(c) Both 1 and 2

(d) Neither 1 nor 2

Solution: B

Eat Right Movement:

• It was launched by the Food Safety and Standards Authority of India (FSSAI).

• The movement aims to cut down salt/sugar and oil consumption by 30% in three years.

• It also aims to engage and enable citizens to improve their health and well-being by making

the right food choices.

36. “Rapid and more inclusive growth” was the main theme of which five year plan?

(a) 12th Five Year Plan

(b) 11th Five Year Plan

(c) 9th Five Year Plan

(d) 10th Five Year Plan

Solution: B

Eleventh Five Year Plan:

1. Its duration was from 2007 to 2012, under the leadership of Manmohan Singh.

2. It was prepared by the C. Rangarajan.

3. Its main theme was “rapid and more inclusive growth”.

4. It achieved a growth rate of 8% against a target of 9% growth.

37. Consider the following statements regarding History of Planning in India

1. National Planning Committee was setup by Indian National Congress in 1938.

2. Peoples Plan was proposed by post war reconstruction Committee of Indian Trade

Union.

3. Sarvodaya Plan in 1950 by Jaiprakash Narayan.

Which of the statements given above is/are correct?

Page 28: INSTA Revision Plan 3.0 - 2020 INSTA Tests€¦ · • State Agricultural Marketing Board made responsible for grading and standardization. 7. Consider the following statements regarding

www.insightsonindia.com 26 INSTA Revision 3.0

INSIGHTSIAS SIMPLYFYING IAS EXAM PREPARATION

(a) 1 and 2 only

(b) 1 and 3 only

(c) 2 and 3 only

(d) 1, 2 and 3

Solution: D

History of Planning in India & Origin of Five-Year Plans

• Though the planned economic development in India began in 1951 with the inception of First

Five Year Plan, theoretical efforts had begun much earlier , even prior to the independence.

Setting up of National Planning Committee by Indian National Congress in 1938, The

Bombay Plan & Gandhian Plan in 1944, Peoples Plan in 1945 (by post war

reconstruction Committee of Indian Trade Union), Sarvodaya Plan in 1950 by Jaiprakash

Narayan were steps in this direction.

38. Consider the following statements regarding Gram Nyayalayas

1. They have both civil and criminal jurisdiction over the offences

2. The pecuniary jurisdiction of the Nyayalayas are fixed by the respective District Courts

3. Gram Nyayadhikari are to be appointed by the State Government in consultation with

the respective High Court.

Which of the statements given above is/are correct?

(a) 1 and 3 only

(b) 2 and 3 only

(c) 1 and 2 only

(d) 1, 2 and 3

Solution: A

Gram Nyayalayas:

• Gram Nyayalayas or village courts are established under the Gram Nyayalayas Act, 2008

for speedy and easy access to justice system in the rural areas of India.

• The Act came into force from 2 October 2009.

Composition:

• The Gram Nyayalayas are presided over by a Nyayadhikari, who will have the same power,

enjoy same salary and benefits of a Judicial Magistrate of First Class. Such Nyayadhikari are

to be appointed by the State Government in consultation with the respective High

Court.

Page 29: INSTA Revision Plan 3.0 - 2020 INSTA Tests€¦ · • State Agricultural Marketing Board made responsible for grading and standardization. 7. Consider the following statements regarding

www.insightsonindia.com 27 INSTA Revision 3.0

INSIGHTSIAS SIMPLYFYING IAS EXAM PREPARATION

Jurisdiction:

• A Gram Nyayalaya have jurisdiction over an area specified by a notification by the State

Government in consultation with the respective High Court.

• The Court can function as a mobile court at any place within the jurisdiction of such Gram

Nyayalaya, after giving wide publicity to that regards.

• They have both civil and criminal jurisdiction over the offences.

• The pecuniary jurisdiction of the Nyayalayas are fixed by the respective High Courts.

• Gram Nyayalayas has been given power to accept certain evidences which would otherwise

not be acceptable under Indian Evidence Act.

Procedure to be followed:

• Gram Nyayalayas can follow special procedures in civil matters, in a manner it deem just and

reasonable in the interest of justice.

• Gram Nyayalayas allow for conciliation of the dispute and settlement of the same in the

first instance.

Appeals:

• Appeal in criminal cases shall lie to the Court of Session, which shall be heard and disposed

of within a period of six months from the date of filing of such appeal.

• Appeal in civil cases shall lie to the District Court, which shall be heard and disposed of within

a period of six months from the date of filing of the appeal.

39. Consider the following statements regarding Five-Year Plans (FYPs)

1. Five-Year Plans (FYPs) are centralized and integrated national economic programs.

2. Joseph Stalin implemented the first FYP in the Soviet Union.

3. For the first eight Plans the emphasis was on a growing public sector with massive

investments in basic and heavy industries.

Which of the statements given above is/are correct?

(a) 1 and 2 only

(b) 1 and 3 only

(c) 2 and 3 only

(d) 1, 2 and 3

Solution: D

Five-Year Plans (FYPs) are centralized and integrated national economic programs. Joseph

Stalin implemented the first FYP in the Soviet Union in the late 1920s. Most communist states

and several capitalist countries subsequently have adopted them.

Page 30: INSTA Revision Plan 3.0 - 2020 INSTA Tests€¦ · • State Agricultural Marketing Board made responsible for grading and standardization. 7. Consider the following statements regarding

www.insightsonindia.com 28 INSTA Revision 3.0

INSIGHTSIAS SIMPLYFYING IAS EXAM PREPARATION

• After independence, India launched its First FYP in 1951, under socialist influence of first

Prime Minister Jawaharlal Nehru. The process began with setting up of Planning Commission

in March 1950 in pursuance of declared objectives of the Government to promote a rapid rise

in the standard of living of the people by efficient exploitation of the resources of the country,

increasing production and offering opportunities to all for employment in the service of the

community. The Planning Commission was charged with the responsibility of making

assessment of all resources of the country, augmenting deficient resources, formulating plans

for the most effective and balanced utilization of resources and determining priorities.

• For the first eight Plans the emphasis was on a growing public sector with massive

investments in basic and heavy industries, but since the launch of the Ninth Plan in

1997, the emphasis on the public sector has become less pronounced and the current thinking

on planning in the country, in general, is that it should increasingly be of an indicative nature.

40. Consider the following statements regarding Union Budget

1. Corporate tax contributes to 25% of income of the government.

2. State’s share of duties and taxes forms the 25% of government expenditure.

Which of the statements given above is/are correct?

(a) 1 only

(b) 2 only

(c) Both 1 and 2

(d) Neither 1 nor 2

Solution: D

Page 31: INSTA Revision Plan 3.0 - 2020 INSTA Tests€¦ · • State Agricultural Marketing Board made responsible for grading and standardization. 7. Consider the following statements regarding

www.insightsonindia.com 29 INSTA Revision 3.0

INSIGHTSIAS SIMPLYFYING IAS EXAM PREPARATION

41. Consider the following statements regarding Indian economy

1. The Indian economy shrank around 24% year-on-year in the second quarter of 2020.

2. In the second quarter of 2020, government consumption decreased by 16%, as the

government implemented relief measures to help curb the impact of the pandemic.

Which of the statements given above is/are correct?

(a) 1 only

(b) 2 only

(c) Both 1 and 2

(d) Neither 1 nor 2

Solution: A

The Indian economy shrank 23.9% year-on-year in the second quarter of 2020, much worse

than market forecasts of an 18.3% drop. It is the biggest contraction on record, as India imposed a

coronavirus lockdown in late March and extended it several times, halting most economic

activities.

On the expenditure side, gross fixed capital formation recorded the biggest decrease (-47.1%).

Private spending shrank 26.7%, inventories fell 20.8%, exports went down 19.8% and imports sank

40.4%. In contrast, government consumption jumped 16.4% as the government implemented relief

measures to help curb the impact of the pandemic.

Page 32: INSTA Revision Plan 3.0 - 2020 INSTA Tests€¦ · • State Agricultural Marketing Board made responsible for grading and standardization. 7. Consider the following statements regarding

www.insightsonindia.com 30 INSTA Revision 3.0

INSIGHTSIAS SIMPLYFYING IAS EXAM PREPARATION

42. World Investment report has been brought out by

(a) World Bank

(b) UNCTAD

(c) World Economic Forum (WEF)

(d) OECD

Solution: B

The World Investment Report has been published annually since 1991 by the United Nations

Conference on Trade and Development (UNCTAD). The report focuses on trends in foreign

direct investment (FDI) worldwide, at the regional and country levels and emerging measures

to improve its contribution.

Other reports published by UNCTAD include Trade and Development Report, The Least

Developed Countries Report, World Economic Situation and Prospects, Information

and Economy Report, Technology and Innovation Report, and Commodities and

Development Report.

43. Consider the following statements regarding Classical Languages

1. Sanskrit was the first language to be declared as Classical language.

2. Odia was the last language to be declared as Classical language.

Which of the statements given above is/are correct?

(a) 1 only

(b) 2 only

(c) Both 1 and 2

Page 33: INSTA Revision Plan 3.0 - 2020 INSTA Tests€¦ · • State Agricultural Marketing Board made responsible for grading and standardization. 7. Consider the following statements regarding

www.insightsonindia.com 31 INSTA Revision 3.0

INSIGHTSIAS SIMPLYFYING IAS EXAM PREPARATION

(d) Neither 1 nor 2

Solution: B

Classical Language in India is an official status within the Republic of India, awarded by the

Government of India.

There are six languages enjoy the ‘Classical’ status:

1. Tamil (declared in 2004).

2. Sanskrit (2005)

3. Kannada (2008)

4. Telugu (2008)

5. Malayalam (2013)

6. Odia (2014)

44. Which of the following public sector enterprises (PSE) are designated with Maharatna

status?

1. Bharat Electronics Limited (BEL)

2. Hindustan Aeronautics Limited (HAL)

3. Steel Authority of India Limited

4. Indian Oil Corporation Limited

Select the correct answer using the code given below

(a) 1, 2 and 3 only

(b) 2 and 4 only

(c) 3 and 4 only

(d) 1, 2, 3 and 4

Solution: C

List of Maharatna, and Navratna CPSEs [As per available information (as on January, 2020)]

Maharatna CPSEs

• Bharat Heavy Electricals

Limited

• Bharat Petroleum

Corporation Limited

• Coal India Limited

• GAIL (India) Limited

• Hindustan Petroleum

Corporation Limited

• Indian Oil Corporation

Limited

• NTPC Limited

• Oil & Natural Gas

Corporation Limited

• Power Grid Corporation

of India Limited

• Steel Authority of India

Limited

Page 34: INSTA Revision Plan 3.0 - 2020 INSTA Tests€¦ · • State Agricultural Marketing Board made responsible for grading and standardization. 7. Consider the following statements regarding

www.insightsonindia.com 32 INSTA Revision 3.0

INSIGHTSIAS SIMPLYFYING IAS EXAM PREPARATION

Navratna CPSEs

• Bharat Electronics

Limited

• Container Corporation of

India Limited

• Engineers India Limited

• Hindustan Aeronautics

Limited

• Mahanagar Telephone

Nigam Limited

• National Aluminium

Company Limited

• NBCC (India) Limited

• NMDC Limited

• NLC India Limited

• Oil India Limited

• Power Finance

Corporation Limited

• Rashtriya Ispat Nigam

Limited

• Rural Electrification

Corporation Limited

• Shipping Corporation of

India Limited

https://dpe.gov.in/about-us/divisions/list-maharatna-navratna-and-miniratna-cpses

45. Consider the following statements regarding Golconda Fort

1. It was originally known as Mankal, and built on a hilltop in the year 1143.

2. The fort was built by the Kakatiya dynasty.

Which of the statements given above is/are not correct?

(a) 1 only

(b) 2 only

(c) Both 1 and 2

(d) Neither 1 nor 2

Solution: D

Golconda Fort was originally known as Mankal, and built on a hilltop in the year 1143. The fort

was built by the Kakatiya dynasty.

• India’s one of the most outstanding citadels, the Golconda fort epitomises the sumptuous

‘Nawabi’ culture of the time.

• The Golconda fort came into the possession of the Bahmani dynasty.

46. Which of the following Centrally Sponsored Schemes is/are categorized as Core of the Core

Schemes?

1. National Social Assistance Progamme

2. Mahatma Gandhi National Rural Employment Guarantee Programme

3. Pradhan Mantri Krishi Sinchai Yojana (PMKSY)

4. National Health Mission (NHM)

Select the correct answer using the code given below:

(a) 1 and 2 only

Page 35: INSTA Revision Plan 3.0 - 2020 INSTA Tests€¦ · • State Agricultural Marketing Board made responsible for grading and standardization. 7. Consider the following statements regarding

www.insightsonindia.com 33 INSTA Revision 3.0

INSIGHTSIAS SIMPLYFYING IAS EXAM PREPARATION

(b) 3 and 4 only

(c) 2 only

(d) 1, 2, 3 and 4

Solution: B

Core of the Core Schemes:

1. National Social Assistance Progamme (Department of Rural Development)

2. Mahatma Gandhi National Rural Employment Guarantee Programme (Department of

Rural Development)

3. Umbrella Scheme for Development of Schedule Castes (Department of Social Justice and

Empowerment)

4. Special Central Assistance

5. Civil Rights, Educational Empowerment, Infrastructure Development, Livelihoods

6. Umbrella Programme for Development of Scheduled Tribes (Ministry of Tribal Affairs)

7. Special Central Assistance

8. Tribal Education

9. Vanbandhu Kalyan Yojana

10. Umbrella Programme for Development of Minorities

11. Umbrella Programme for Development of Other Vulnerable Groups Pradhan Mantri Krishi

Sinchai Yojana (PMKSY) and National Health Mission (NHM) are categorized as Core

schemes.

47. Consider the following statements regarding the National Investment Fund (NIF):

1. It will have proceeds from disinvestment of Central Public Sector Enterprises.

2. The corpus of NIF was to be of a permanent nature.

3. It will be used for recapitalization of public sector banks and public sector insurance

companies so as to strengthen them by further capital infusion towards achieving the

Basel III norms

Which of the statements given above is/are correct?

(a) 1 and 2 only

(b) 2 and 3 only

(c) 1 and 3 only

(d) 1, 2 and 3

Page 36: INSTA Revision Plan 3.0 - 2020 INSTA Tests€¦ · • State Agricultural Marketing Board made responsible for grading and standardization. 7. Consider the following statements regarding

www.insightsonindia.com 34 INSTA Revision 3.0

INSIGHTSIAS SIMPLYFYING IAS EXAM PREPARATION

Solution: D

National Investment Fund

• Government had constituted the National Investment Fund (NIF) in November, 2005 into

which the proceeds from disinvestment of Central Public Sector Enterprises were to be

channelized. The corpus of NIF was to be of a permanent nature and NIF was to be

professionally managed to provide sustainable returns to the Government, without depleting

the corpus. Selected Public Sector Mutual Funds, namely UTI Asset Management Company

Ltd., SBI Funds Management Private Ltd. and LIC Mutual Fund Asset Management

Company Ltd. were entrusted with the management of the NIF corpus.

• As per this Scheme, 75% of the annual income of the NIF was to be used for financing selected

social sector schemes which promote education, health and employment. The residual 25% of

the annual income of NIF was to be used to meet the capital investment requirements of

profitable and revivable PSUs.

• In view of the difficult economic situation caused by the global slowdown of 2008-09 and a

severe drought in 2009-10, Government approved a change in the policy for utilization of

disinvestment proceeds (5th of November 2009) by granting a one-time exemption to utilize

the disinvestment proceeds directly for selected Social Sector Schemes allocated by

Department of Expenditure/ Planning Commission. This exemption was to be operational for

the period April 2009-March 2012. In view of the persistent difficult condition of the economy,

the exemption from channelizing the disinvestment proceeds in the NIF was further extended

by another year i.e. from April 2012 to March 2013

In order to align the NIF with the disinvestment Policy, Government decided (17th January 2013)

that the disinvestment proceeds, with effect from the fiscal year 2013-14, will be credited to the

existing NIF which is a ‘Public Account’ under the Government Accounts and the funds

would remain there until withdrawn/invested for the approved purposes. It was also

simultaneously decided that the NIF would be utilized for the following purposes:

1. Subscribing to the shares being issued by the CPSE on rights basis so as to ensure that

51% ownership of the Government in CPSEs is not diluted.

2. Preferential allotment of shares of the CPSE to promoters as per SEBI (Issue of Capital

and Disclosure Requirements) Regulations, 2009 so that Government shareholding does

not go down below 51% in all cases where the CPSEs desire to raise fresh equity to meet

their Capex programme.

3. Recapitalization of public sector banks and public sector insurance companies so as to

strengthen them by further capital infusion towards achieving the Basel III norms

The Government further approved inclusion of the following purposes also, to be financed from the

NIF (21st February, 2013).

1. Investment by Government in RRBs/IIFCL/NABARD/Exim Bank;.

2. Equity infusion in various Metro projects;

3. Investment in Bhartiya Nabhikiya Vidyut Nigam Limited and Uranium Corporation of

India Ltd.

4. Investment in Indian Railways towards capital expenditure.

Page 37: INSTA Revision Plan 3.0 - 2020 INSTA Tests€¦ · • State Agricultural Marketing Board made responsible for grading and standardization. 7. Consider the following statements regarding

www.insightsonindia.com 35 INSTA Revision 3.0

INSIGHTSIAS SIMPLYFYING IAS EXAM PREPARATION

48. Consider the following statements regarding International Solar Alliance (ISA)

1. Nations lies in tropical region are only eligible to become the member of the

organization.

2. Headquarter of the organization is located in Paris, France.

Which of the statements given above is/are correct?

(a) 1 only

(b) 2 only

(c) Both 1 and 2

(d) Neither 1 nor 2

Solution: D

The International Solar Alliance is an alliance of 121 countries initiated by India, most of them

being sunshine countries, which lie either completely or partly between the Tropic of Cancer and

the Tropic of Capricorn.

• It was launched at the Paris Climate Change Summit (COP21) in 2015. Hence,

• Membership of the body is open to all countries that want to join the grouping, with no

restrictions on duration of sunlight or geographical location.

• Headquarter of the organization is located in Gurugram, India.

49. After liberalization, India has undergone structural change in its economy. In this context,

‘structural change’ necessarily implies

(a) Change in the production pattern of agriculture

(b) Change in the contribution of different sector of economy

(c) Change in the production technology of basic goods

(d) Change in demographic characteristics

Solution: B

Structure of an economy refers to the fundamental features of the economy like the size

of the primary, secondary and tertiary sectors in terms of their contribution to GDP and

employment. Other important elements of structure are trade composition (the items that we

export and import), saving GDP ratio (level of savings as a percent of GDP) etc. Structure of the

economy thus means the occupational structure, sectoral distribution of income, industrial pattern,

composition of exports, saving- GDP ratio etc.

Page 38: INSTA Revision Plan 3.0 - 2020 INSTA Tests€¦ · • State Agricultural Marketing Board made responsible for grading and standardization. 7. Consider the following statements regarding

www.insightsonindia.com 36 INSTA Revision 3.0

INSIGHTSIAS SIMPLYFYING IAS EXAM PREPARATION

50. Which among the following countries announced the formation of the Strategic Partnership

Council with India?

(a) Oman

(b) Saudi Arabia

(c) Iraq

(d) Yemen

Solution: B

Agreement for the Strategic Partnership Council (SPC) between India and Saudi Arabia

has been signed.

India will become the fourth country with which the Kingdom has formed a strategic

partnership, the others being the UK, France and China.

The SPC will have two parallel tracks: Political, security, culture and society, headed by both

countries’ foreign ministers; and economy and investment, headed by India’s commerce and

industry minister and the Saudi energy minister.

DAY – 33

51. Consider the following statements regarding Dividend Distribution Tax

1. The dividend is the part of profits that the company shares with its shareholders.

2. Under the Income Tax Act, any domestic firm which is declaring or distributing dividend

has to pay DDT at the rate of 15 per cent on the gross amount of dividend.

Which of the statements given above is/are correct?

(a) 1 only

(b) 2 only

(c) Both 1 and 2

(d) Neither 1 nor 2

Solution: C

Dividend Distribution Tax

• It is a tax levied on dividends that a company pays to its shareholders out of its

profits.

• The Dividend Distribution Tax, or DDT, is taxable at source, and is deducted at the time of

the company distributing dividends.

Page 39: INSTA Revision Plan 3.0 - 2020 INSTA Tests€¦ · • State Agricultural Marketing Board made responsible for grading and standardization. 7. Consider the following statements regarding

www.insightsonindia.com 37 INSTA Revision 3.0

INSIGHTSIAS SIMPLYFYING IAS EXAM PREPARATION

• The dividend is the part of profits that the company shares with its shareholders.

• The law provides for the Dividend Distribution Tax to be levied at the hands of the

company, and not at the hands of the receiving shareholder.

• However, an additional tax is imposed on the shareholder, who receives over Rs. 10 lakh

in dividend income in a financial year.

Is Dividend Distribution Tax applicable to private companies?

• Under Section 115-O, the Income Tax Act, any domestic firm which is declaring or distributing

dividend has to pay DDT at the rate of 15 per cent on the gross amount of dividend.

Is Dividend Distribution Tax fair?

• Market participants, especially brokers, have been calling for long to scrap the DDT. The tax

makes markets unattractive as it leads to significant taxation of corporate earnings, they

argue.

• Other than Dividend Distribution Tax (DDT), the Securities Transaction Tax (STT) and Long-

Term Capital Gains (LTCG) tax are other major taxes levied on market instruments.

52. Consider the following statements regarding cash reserve ratio (CRR)

1. It is a certain minimum amount of deposit that the commercial banks have to hold as

reserves with the central bank.

2. At the time of high inflation, RBI decrease the Cash Reserve Ratio and curbs excess flow

of money in the economy.

3. Banks do not earn any interest for maintaining CRR with the RBI.

Which of the statements given above is/are correct?

(a) 1 and 2 only

(b) 1 and 3 only

(c) 2 and 3 only

(d) 1, 2 and 3

Solution: B

The Reserve Bank of India (RBI) has exempted banks from maintaining cash reserve ratio

(CRR) for loans to retail and micro, small and medium enterprises for five years, if these

loans are extended between January 31 and July 31, 2020.

Background:

• At present, CRR is 4% of net demand and time liabilities. Banks do not earn any interest

for maintaining CRR with the RBI.

What is CRR?

• It is a certain minimum amount of deposit that the commercial banks have to hold as reserves

with the central bank.

Page 40: INSTA Revision Plan 3.0 - 2020 INSTA Tests€¦ · • State Agricultural Marketing Board made responsible for grading and standardization. 7. Consider the following statements regarding

www.insightsonindia.com 38 INSTA Revision 3.0

INSIGHTSIAS SIMPLYFYING IAS EXAM PREPARATION

• The percentage of cash required to be kept in reserves, vis-a-vis a bank’s total deposits, is

called the Cash Reserve Ratio.

• The cash reserve is either stored in the bank’s vault or is sent to the RBI. Banks do not get

any interest on the money that is with the RBI under the CRR requirements.

There are two primary purposes of the Cash Reserve Ratio:

• Since a part of the bank’s deposits is with the Reserve Bank of India, it ensures the security

of the amount. It makes it readily available when customers want their deposits back.

• Also, CRR helps in keeping inflation under control. At the time of high inflation in the

economy, RBI increases the CRR, so that banks need to keep more money in reserves so that

they have less money to lend further.

How does Cash Reserve Ratio help in times of high inflation?

• At the time of high inflation, the government needs to ensure that excess money is not

available in the economy.

• To that extent, RBI increases the Cash Reserve Ratio, and the amount of money that

is available with the banks reduces. This curbs excess flow of money in the economy.

When the government needs to pump funds into the system, it lowers the CRR rate, which

in turn, helps the banks provide loans to a large number of businesses and industries for

investment purposes. Lower CRR also boosts the growth rate of the economy.

53. Which of the following reports is/are published by UNCTAD?

1. Trade and Development Report

2. World Investment Report

3. Technology and Innovation Report

4. Digital Economy Report

Select the correct answer using the code given below:

(a) 1, 2 and 3 only

(b) 2, 3 and 4 only

(c) 1, 3 and 4 only

(d) 1, 2, 3 and 4

Solution: D

United Nations Conference on Trade and Development (UNCTAD):

• UNCTAD is a permanent intergovernmental body established by the United Nations General

Assembly in 1964.

• It is part of the UN Secretariat. It reports to the UN General Assembly and the Economic

and Social Council, but has its own membership, leadership, and budget. It is also a part of

the United Nations Development Group.

Page 41: INSTA Revision Plan 3.0 - 2020 INSTA Tests€¦ · • State Agricultural Marketing Board made responsible for grading and standardization. 7. Consider the following statements regarding

www.insightsonindia.com 39 INSTA Revision 3.0

INSIGHTSIAS SIMPLYFYING IAS EXAM PREPARATION

Objectives and roles:

• It supports developing countries to access the benefits of a globalized economy more fairly

and effectively. Along with other UN departments and agencies, it also measures the progress

made in the Sustainable Development Goals, as set out in Agenda 2030.

Reports published by UNCTAD are:

1. Trade and Development Report

2. World Investment Report

3. Technology and Innovation Report

4. Digital Economy Report

54. Consider the following statements regarding targeted long-term repo operation (TLTRO)

1. The LTRO is a tool under which the central bank provides one-month to three-month

money to banks at the prevailing repo rate.

2. LTRO operations are intended to prevent short-term interest rates in the market from

drifting a long way away from the policy rate.

Which of the statements given above is/are correct?

(a) 1 only

(b) 2 only

(c) Both 1 and 2

(d) Neither 1 nor 2

Solution: B

• The Reserve Bank of India (RBI) has said it has received Rs 1.13 lakh crore worth of bids in

the targeted long term repo operation (TLTRO) conducted for an amount of Rs 25,000

crore with a three-year tenor.

• The RBI received 18 bids in the auction. The total bids that were received amounted to Rs

1.13 lakh crore, implying a bid to cover ratio — the number of bids received relative to the

notified amount — of 4.5.

What is LTRO?

• The LTRO is a tool under which the central bank provides one-year to three-year money

to banks at the prevailing repo rate, accepting government securities with matching

or higher tenure as the collateral.

How is it different from LAF and MSF?

• While the RBI’s current windows of liquidity adjustment facility (LAF) and marginal

standing facility (MSF) offer banks money for their immediate needs ranging from 1-28

days, the LTRO supplies them with liquidity for their 1- to 3-year needs. LTRO operations are

Page 42: INSTA Revision Plan 3.0 - 2020 INSTA Tests€¦ · • State Agricultural Marketing Board made responsible for grading and standardization. 7. Consider the following statements regarding

www.insightsonindia.com 40 INSTA Revision 3.0

INSIGHTSIAS SIMPLYFYING IAS EXAM PREPARATION

intended to prevent short-term interest rates in the market from drifting a long way away

from the policy rate, which is the repo rate.

Why is it important?

• As banks get long-term funds at lower rates, their cost of funds falls.

• In turn, they reduce interest rates for borrowers.

• LTRO helped RBI ensure that banks reduce their marginal cost of funds-based lending rate,

without reducing policy rates.

• LTRO also showed the market that RBI will not only rely on revising repo rates and

conducting open market operations for its monetary policy, but also use new tools to achieve

its intended objectives.

55. Consider the following statements regarding Hangul

1. It is the state animal of Jammu & Kashmir.

2. It is restricted to the Dachigam National Park.

3. The IUCN’s Red List has classified it as Endangered species.

Which of the statements given above is/are correct?

(a) 1 and 2 only

(b) 2 and 3 only

(c) 1 and 3 only

(d) 1, 2 and 3

Solution: A

Hangul:

• It is the state animal of Jammu & Kashmir.

• It is restricted to the Dachigam National Park some 15 km north-west of Jammu &

Kashmir’s summer capital Srinagar.

• It is placed under Schedule I of the Indian Wildlife (Protection) Act, 1972 and the J&K

Wildlife Protection Act, 1978.

• The Hangul was once widely distributed in the mountains of Kashmir and parts of Chamba

district in neighbouring Himachal Pradesh.

• The IUCN’s Red List has classified it as Critically Endangered and is similarly listed under

the Species Recovery Programme of the Wildlife Institute of India (WII) and the

Environmental Information System (ENVIS) of the MoEFCC.

• A massive decline in the population of Kashmir’s iconic wildlife species, the Hangul

(Cervus hanglu hanglu), also known as the Kashmir stag, continues to be a big concern.

Page 43: INSTA Revision Plan 3.0 - 2020 INSTA Tests€¦ · • State Agricultural Marketing Board made responsible for grading and standardization. 7. Consider the following statements regarding

www.insightsonindia.com 41 INSTA Revision 3.0

INSIGHTSIAS SIMPLYFYING IAS EXAM PREPARATION

56. Consider the following statements regarding Helicopter Money

1. It involves printing large sums of money and distributing it to the public.

2. The direct impact of Helicopter Money is rise in disposable incomes of the people.

Which of the statements given above is/are correct?

(a) 1 only

(b) 2 only

(c) Both 1 and 2

(d) Neither 1 nor 2

Solution: C

• Amid rising concerns over economic crisis that has been triggered by the COVID-19 lockdown;

Helicopter Money is one concept that is being considered by authorities’ world over.

• In fact, Telangana Chief Minister K. Chandrashekar Rao has suggested RBI to adopt the

concept of Helicopter Money to help state governments tide over the current crisis and

kickstart economic activity in India.

What is helicopter money?

• This is an unconventional monetary policy tool aimed at bringing a flagging economy

back on track. It involves printing large sums of money and distributing it to the public.

American economist Milton Friedman coined this term.

Why it is called so?

• It basically denotes a helicopter dropping money from the sky. Friedman used the term to

signify “unexpectedly dumping money onto a struggling economy with the intention to shock

it out of a deep slump.” Under such a policy, a central bank “directly increase the money supply

and, via the government, distribute the new cash to the population with the aim of boosting

demand and inflation.”

Why is helicopter money in news now?

• With the coronavirus-hit economy falling deeper and deeper into a chasm with each passing

day, Telangana chief minister KC Rao has said helicopter money can help states comes out of

this morass. He asked for the release of 5% funds from GDP by way of quantitative easing

(QE).

Is helicopter money the same as quantitative easing?

• Quantitative easing also involves the use of printed money by central banks to buy

government bonds. But not everyone views the money used in QE as helicopter money. It sure

means printing money to monetise government deficits, but the govt has to pay back for the

assets that the central bank buys. It’s not the same as bond-buying by central banks “in which

bank-owned assets are swapped for new central bank reserves.”

How will Helicopter Money help Indian Economy?

• Simply put, Helicopter Money means extension of non-repayable money transfer from

the central bank to the state and central governments, to infuse liquidity in the system.

Page 44: INSTA Revision Plan 3.0 - 2020 INSTA Tests€¦ · • State Agricultural Marketing Board made responsible for grading and standardization. 7. Consider the following statements regarding

www.insightsonindia.com 42 INSTA Revision 3.0

INSIGHTSIAS SIMPLYFYING IAS EXAM PREPARATION

• The policy aims at putting more money into the pockets of people to nudge them to spend more

money and in turn pick-up economic activity in the country.

• The direct impact of Helicopter Money is rise in disposable incomes of the people,

increase in money supply with an intention to boost demand and inflation in the

economy.

57. Which of the following entities are eligible for appointment of Business Correspondents

(BCS) for banks?

1. NGOs/MFls set up under Societies/Trust Acts

2. Post offices

3. Retired government employees

4. Fair price shop owners

Select the correct answer using the code given below:

(a) 1, 2 and 3 only

(b) 2, 3 and 4 only

(c) 1, 2 and 4 only

(d) 1, 2, 3 and 4

Solution: D

Business Correspondent (BC) – An Introduction

• Business correspondents are bank representatives. They help villagers to open bank

accounts. Business Correspondents get commission from bank for every new account

opening, every transaction made via them, every loan-application processed etc.

• The Business Correspondent carries a mobile device and helps villagers in banking

transactions. (Deposit money, take money out of savings account, loans etc.). The villager gives

his thumb impression or electronic signature, and gets the money.

Eligibility to become a Business Correspondent

As per the RBI guidelines, the following entities are eligible for appointment of Business

Correspondents (BCS) for banks:

1. NGOs/MFls set up under Societies/Trust Acts

2. Societies registered under Mutually Aided Cooperative Societies Acts or the Cooperative

Societies Acts of States

3. Section 25 companies that are standalone entities or in which NBFCs, banks, telecom

companies and other corporate entities or their holding companies did not have equity

holdings in excess of 10 per cent

4. Post offices

5. Retired bank employees,

Page 45: INSTA Revision Plan 3.0 - 2020 INSTA Tests€¦ · • State Agricultural Marketing Board made responsible for grading and standardization. 7. Consider the following statements regarding

www.insightsonindia.com 43 INSTA Revision 3.0

INSIGHTSIAS SIMPLYFYING IAS EXAM PREPARATION

6. Ex-servicemen

7. Retired government employees.

8. Individual kirana / medical / fair price shop owners Individual Public Call Office (PCO)

operators

9. Agents of Small Savings Schemes of Government of India/Insurance Companies

Individuals who own petrol pumps

10. Retired teachers

11. Authorised functionaries of well run Self Help Groups (SHGs) linked to banks Non deposit

taking NBFCs (non-banking finance companies) in the nature of loan companies whose

micro finance portfolio is not less than 80 per cent of their loan outstanding in the

financially excluded districts as identified by the Committee on Financial Inclusion.

RBI has now permitted banks to engage any individual, including those operating Common

Service Centres (CSCs) as BC, subject to banks’ comfort level and their carrying out suitable

due diligence as also instituting additional safeguards as may be considered appropriate to

minimise the agency risks.

58. Consider the following statements regarding North East Rural Livelihood Project

1. It is an IMF aided project.

2. It aims to improve rural livelihoods especially that of women, unemployed youth and

the most disadvantaged, in all the northeastern states.

Which of the statements given above is/are correct?

(a) 1 only

(b) 2 only

(c) Both 1 and 2

(d) Neither 1 nor 2

Solution: D

North East Rural Livelihood Project (NERLP):

• A study finds that North East Rural Livelihood Project (NERLP) improves livelihoods of

300,000 households in 11 districts of Mizoram, Nagaland, Tripura and Sikkim.

• It is a World Bank aided, multi-state livelihood project under the Ministry of Development of

North Eastern Region (DoNER), launched in 2012.

• Implemented in 11 districts of Mizoram, Nagaland, Tripura and Sikkim.

• Aim: to improve rural livelihoods especially that of women, unemployed youth and the most

disadvantaged, in four North Eastern States.

Page 46: INSTA Revision Plan 3.0 - 2020 INSTA Tests€¦ · • State Agricultural Marketing Board made responsible for grading and standardization. 7. Consider the following statements regarding

www.insightsonindia.com 44 INSTA Revision 3.0

INSIGHTSIAS SIMPLYFYING IAS EXAM PREPARATION

• The project has focussed on five development strategies, namely, social empowerment,

economic empowerment, partnership development, project management and livelihood &

value chain developments.

59. Consider the following statements regarding Regional Rural Banks

1. Regional Rural Banks were set up on the basis of the recommendations of the

Narasimham Working Group (1975).

2. Regional Rural Banks are established under Reserve Bank of India Act, 1934.

3. The equity of a regional rural bank is held by the Central Government, concerned State

Government and the Sponsor Bank in the proportion of 50:15:35.

Which of the statements given above is/are correct?

(a) 1 and 2 only

(b) 1 and 3 only

(c) 2 and 3 only

(d) 1, 2 and 3

Solution: B

What are RRBs?

• Regional Rural Banks were set up on the basis of the recommendations of the Narasimham

Working Group (1975), and after the legislation of the Regional Rural Banks Act, 1976.

• The first Regional Rural Bank “Prathama Grameen Bank” was set up on 2nd October, 1975.

• The equity of a regional rural bank is held by the Central Government, concerned State

Government and the Sponsor Bank in the proportion of 50:15:35.

• The Regional Rural Banks (RRBs) were established in 1975 under the provisions of the

Ordinance promulgated on 26th September, 1975 and Regional Rural Banks Act, 1976 with a

view to developing the rural economy by providing, for the purpose of development of

agriculture, trade, commerce, industry and other productive activities in the rural areas,

credit and other facilities, particularly to small and marginal farmers, agricultural labourers,

artisans and small entrepreneurs, and for matters connected therewith and incidental thereto.

60. Consider the following statements regarding Lok Sabha Speaker

1. He is nominated by the President.

2. The Speaker of Lok Sabha comes at sixth position in the Order of Precedence of

Government of India.

3. The Speaker of Lok Sabha leaves the office immediately after dissolution of the

assembly.

Which of the statements given above is/are correct?

Page 47: INSTA Revision Plan 3.0 - 2020 INSTA Tests€¦ · • State Agricultural Marketing Board made responsible for grading and standardization. 7. Consider the following statements regarding

www.insightsonindia.com 45 INSTA Revision 3.0

INSIGHTSIAS SIMPLYFYING IAS EXAM PREPARATION

(a) 2 only

(b) 2 and 3 only

(c) 1 and 3 only

(d) 3 only

Solution: A

Speaker of the Lok Sabha:

• The chairman or the Presiding Officer of Lok Sabha is called Speaker.

• The speaker of the Lok Sabha is elected from all other members by simple majority.

• Any member of Parliament is eligible to be nominated as a speaker but most commonly the

candidate of ruling party or the party with majority wins this post.

• However, there are certain cases when the elected Speaker does not belonged to the majority

ruling party of Lok Sabha (G. M. C. Balyogi, Manohar Joshi, Somnath Chatterjee).

Functions and Powers of Lok Sabha Speakers:

• Speaker of Lok Sabha is basically the head of the house and presides over the sittings of

Parliament and controls its working.

• The constitution has tried to ensure the independence of Speaker by charging his salary on

the consolidated Fund of India and the same is not subject to vote of Parliament.

• While debating or during general discussion on a bill, the members of the parliament have to

address only to the Speaker.

• Whenever there is a joint sitting of both houses of Parliament (Lok Sabha & Rajya Sabha) the

Speaker of the Lok Sabha presides over this meeting.

• The Speaker of Lok Sabha comes at sixth position in the Order of Precedence of Government

of India.

• In the normal circumstances the Speaker does not casts his vote over any matter in Lok Sabha.

But when ever there is a tie on votes between the ruling party and opposition, the Speaker at

that time can exercise his vote.

• It is the Speaker who decides the agenda of various discussions.

• The speaker has the power to adjourn or suspend the house/meetings if the quorum is not met.

• The Speaker ensures the discipline and decorum of the house. If the speaker finds the

behaviour and a member of Parliament is not good, he/she can punish the unruly members by

suspending.

• The Speaker decides weather a bill brought to the house is a money bill or not. In the case

Speaker decides some bill as a money bill, this decision can not be challenged.

• Speaker is the final and sole authority to allow different types of motions and resolutions such

as No Confidence Motion, Motion of Adjournment, Censure Motion etc.

Page 48: INSTA Revision Plan 3.0 - 2020 INSTA Tests€¦ · • State Agricultural Marketing Board made responsible for grading and standardization. 7. Consider the following statements regarding

www.insightsonindia.com 46 INSTA Revision 3.0

INSIGHTSIAS SIMPLYFYING IAS EXAM PREPARATION

• The Speaker of Lok Sabha does not leave the office just after dissolution of the assembly. He

continues to be in the office till the newly formed assembly takes its first meeting and elects

the new Speaker.

The Speaker of Lok Sabha automatically disqualifies from his post if:

• he is no longer the Member of Parliament.

• if he tenders his resignation to the Deputy Speaker.

• if he holds the office of profit under central government or any state government.

• if he is of unsound mind and that too declared by the court of law.

• if he is declared undischarged insolvent.

• if he is no longer the citizen of India or voluntarily accepts the citizenship of any other country.

• if he is removed from the post of Speaker by passing a resolution by majority of the members

of Lok Sabha. This is to note that during resolution for removal of Speaker, the Speaker is not

in position to cast his vote even if there is tie.

61. Consider the following statements regarding Payment Infrastructure Development Fund

(PIDF)

1. It is setup by Department of Financial Services.

2. This fund has been created to encourage acquirers to deploy point of sale (PoS)

infrastructure in tier-3 to tier-6 centres and north eastern states.

3. The fund will be managed and administered by the RBI.

Which of the statements given above is/are correct?

(a) 1 and 2 only

(b) 1 and 3 only

(c) 2 and 3 only

(d) 1, 2 and 3

Solution: C

• In an effort to give a push to digital payments across the country, the Reserve Bank of India

(RBI) is setting up a Payment Infrastructure Development Fund (PIDF) of Rs 500

crore.

All you need to know about the fund:

• Objective: This fund has been created to encourage acquirers to deploy point of sale (PoS)

infrastructure, both physical and digital, in tier-3 to tier-6 centres and north eastern states.

• Contributions to the fund: The RBI has made an initial contribution of Rs 250 crore

covering half the fund. The remaining will come from the card issuing banks and card

networks operating in the country.

Page 49: INSTA Revision Plan 3.0 - 2020 INSTA Tests€¦ · • State Agricultural Marketing Board made responsible for grading and standardization. 7. Consider the following statements regarding

www.insightsonindia.com 47 INSTA Revision 3.0

INSIGHTSIAS SIMPLYFYING IAS EXAM PREPARATION

• Management: The fund will be governed through an advisory council but it will be managed

and administered by the RBI.

62. Consider the following statements regarding Wholesale Price Index

1. The numbers are released by the Central Statistics Office.

2. RBI uses WPI as a key measure of inflation for policy purposes.

Which of the statements given above is/are correct?

(a) 1 only

(b) 2 only

(c) Both 1 and 2

(d) Neither 1 nor 2

Solution: D

Wholesale Price Index, or WPI, measures the changes in the prices of goods sold and traded in

bulk by wholesale businesses to other businesses. WPI is unlike the Consumer Price Index

(CPI), which tracks the prices of goods and services purchased by consumers.

• The numbers are released by the Economic Advisor in the Ministry of Commerce and

Industry. An upward surge in the WPI print indicates inflationary pressure in the economy

and vice versa. The quantum of rise in the WPI month-after-month is used to measure the

level of wholesale inflation in the economy.

• Even as the WPI is used as a key measure of inflation in some economies, the RBI no longer

uses it for policy purposes, including setting repo rates. The central bank currently uses CPI

or retail inflation as a key measure of inflation to set the monetary and credit policy.

• The new series of Wholesale Price Index (WPI) with base 2011-12 is effective from April

2017.

63. West Asia peace plan term of seen in the news, is unveiled by

(a) India

(b) Israel

(c) Japan

(d) United States of America

Solution: D

Page 50: INSTA Revision Plan 3.0 - 2020 INSTA Tests€¦ · • State Agricultural Marketing Board made responsible for grading and standardization. 7. Consider the following statements regarding

www.insightsonindia.com 48 INSTA Revision 3.0

INSIGHTSIAS SIMPLYFYING IAS EXAM PREPARATION

• West Asia peace plan- The West Asia peace plan was recently unveiled by U.S. President

Trump. It plans to revive the stalled two-state talks between the Israelis and the Palestinians.

It seeks to give the Israelis an expansive state with Jerusalem as its “undivided capital” and

tight security control over a future Palestinian state.

64. Which of the following are categories under priority sector?

1. Export Credit

2. Renewable Energy

3. Education

4. Agriculture

Select the correct answer using the code given below:

(a) 1, 2 and 3 only

(b) 2, 3 and 4 only

(c) 1, 2 and 4 only

(d) 1, 2, 3 and 4

Solution: D

What are the different categories under priority sector?

Priority Sector includes the following categories:

(i) Agriculture

(ii) Micro, Small and Medium Enterprises

(iii) Export Credit

(iv) Education

(v) Housing

(vi) Social Infrastructure

(vii) Renewable Energy

(viii) Others

What is included under Weaker Sections under priority sector?

Priority sector loans to the following borrowers are eligible to be considered under Weaker Sections

category:-

No. Category

1. Small and Marginal Farmers

2. Artisans, village and cottage industries where individual credit limits do not exceed ₹

0.1 million

3. Beneficiaries under Government Sponsored Schemes such as National Rural Livelihoods

Mission (NRLM), National Urban Livelihood Mission (NULM) and Self Employment

Scheme for Rehabilitation of Manual Scavengers (SRMS)

4. Scheduled Castes and Scheduled Tribes

5. Beneficiaries of Differential Rate of Interest (DRI) scheme

6. Self Help Groups

7. Distressed farmers indebted to non-institutional lenders

8. Distressed persons other than farmers, with loan amount not exceeding ₹ 0.1 million per

borrower to prepay their debt to non-institutional lenders

9. Individual women beneficiaries up to ₹ 0.1 million per borrower

Page 51: INSTA Revision Plan 3.0 - 2020 INSTA Tests€¦ · • State Agricultural Marketing Board made responsible for grading and standardization. 7. Consider the following statements regarding

www.insightsonindia.com 49 INSTA Revision 3.0

INSIGHTSIAS SIMPLYFYING IAS EXAM PREPARATION

10. Persons with disabilities

11. Overdraft limit to PMJDY account holder upto ₹ 10,000/- with age limit of 18-65 years

12. Minority communities as may be notified by Government of India from time to time

65. Consider the following statements regarding Judicial Review

1. The power of judicial review can be curtailed or excluded through a constitutional

amendment.

2. In India, the Constitution confers the power of judicial review on both Supreme Court

and High Court.

Which of the statements given above is/are correct?

(a) 1 only

(b) 2 only

(c) Both 1 and 2

(d) Neither 1 nor 2

Solution: B

Judicial review is the power of the judiciary to examine the constitutionality of legislative

enactments and executive orders of both the Central and State governments.

• The Supreme Court has declared the power of judicial review as a basic feature of the

Constitution or an element of the basic structure of the Constitution. Hence, the power of

judicial review cannot be curtailed or excluded even by a constitutional amendment.

• The doctrine of judicial review originated and developed in the USA. In India, the Constitution

confers the power of judicial review on the judiciary (both the Supreme Court as well as High

Courts).

66. Which of the following category has the lowest weightage in Index of Eight Core Industries?

(a) Coal

(b) Steel

(c) Fertilizers

(d) Cement

Solution: C

Page 52: INSTA Revision Plan 3.0 - 2020 INSTA Tests€¦ · • State Agricultural Marketing Board made responsible for grading and standardization. 7. Consider the following statements regarding

www.insightsonindia.com 50 INSTA Revision 3.0

INSIGHTSIAS SIMPLYFYING IAS EXAM PREPARATION

The Office of Economic Adviser, Department for Promotion of Industry and Internal Trade is

releasing Index of Eight Core Industries.

Sector Weight

Coal 10.3335

Crude Oil 8.9833

Natural Gas 6.8768

Refinery Products 28.0376

Fertilizers 2.6276

Steel 17.9166

Cement 5.3720

Electricity 19.8530

67. Which of the following constitute the largest component of India’s Foreign Exchange

Reserves?

(a) Foreign currency assets (FCAs)

(b) India's reserve position with the International Monetary Fund (IMF)

(c) Gold deposits

(d) Special Drawing Rights

Solution: A

• India’s forex reserves comprise Foreign Currency Assets (FCAs), gold reserves, Special

Drawing Rights (SDRs) and India’s reserve position with the International Monetary

Fund (IMF).

• Foreign currency assets (FCAs) are the largest component of the forex reserves.

68. Consider the following statements regarding Lok Adalat

1. Lok Adalats have been given statutory status under the Legal Services Authorities Act,

1987.

2. A permanent Lok Adalat can pass an award on merits, even without the consent of

parties.

Which of the statements given above is/are correct?

(a) 1 only

(b) 2 only

(c) Both 1 and 2

(d) Neither 1 nor 2

Solution: C

Page 53: INSTA Revision Plan 3.0 - 2020 INSTA Tests€¦ · • State Agricultural Marketing Board made responsible for grading and standardization. 7. Consider the following statements regarding

www.insightsonindia.com 51 INSTA Revision 3.0

INSIGHTSIAS SIMPLYFYING IAS EXAM PREPARATION

Lok Adalat is a forum where the cases (or disputes) which are pending in a court or which are at

pre-litigation stage (not yet brought before a court) are compromised or settled in an amicable

manner.

• Lok Adalats have been given statutory status under the Legal Services Authorities Act,

1987. Under the said Act, the award (decision) made by the Lok Adalats is deemed to be a

decree of a civil court and is final and binding on all parties and no appeal against such an

award lies before any court of law.

• A permanent Lok Adalat can pass an award on merits, even without the consent of parties.

Such an award is final and binding. From that no appeal is possible.

69. Stagflation is a condition of

(a) Low level of Unemployment and High level of growth

(b) Low level of Unemployment and Low level of growth

(c) High level of Unemployment and High level of growth

(d) High level of Unemployment and Low level of growth

Solution: D

Stagflation is a condition of slow economic growth and relatively high unemployment, or

economic stagnation, accompanied by rising prices, or inflation.

70. Which of the following organization releases Word Economic Situation Report?

(a) International Monetary Fund

(b) World Bank

(c) Organization for Economic Co-operation and Development

(d) United Nations

Solution: D

World Economic Situation Prospects (WESP) is a joint product of the United Nations

Department of Economic and Social Affairs (UN DESA), the United Nations

Conferenceon Trade and Development (UNCTAD) and the five United Nations regional

commissions;

The Indian economy is expected to register a GDP growth of 5.7 per cent this fiscal, according to

the newly launched UN World Economic Situation and Prospects (UNWESP) report 2020.

Page 54: INSTA Revision Plan 3.0 - 2020 INSTA Tests€¦ · • State Agricultural Marketing Board made responsible for grading and standardization. 7. Consider the following statements regarding

www.insightsonindia.com 52 INSTA Revision 3.0

INSIGHTSIAS SIMPLYFYING IAS EXAM PREPARATION

71. Consider the following statements regarding the Transfer payments:

1. They are receipts which the residents of a country receive by providing certain goods or

services in return.

2. They could be given by the government or by private citizens living abroad

Which of the statements given above is/are correct?

(a) 1 only

(b) 2 only

(c) Both 1 and 2

(d) Neither 1 nor 2

Solution: B

Transfer payments are the receipts which the residents of a country get for ‘free’, without

having to provide any goods or services in return.

• They consist of gifts, remittances and grants.

• They could be given by the government or by private citizens living abroad.

72. Consider the following statements regarding the Minimum Alternate Tax,:

1. It is an indirect tax.

2. It is applicable to both private and public corporate entities.

Which of the statements given above is/are correct?

(a) 1 only

(b) 2 only

(c) Both 1 and 2

(d) Neither 1 nor 2

Solution: B

Minimum Alternate Tax (MAT) is a tax effectively introduced in India by the Finance Act of

1987, vide Section 115J of the Income Tax Act, 1961 (IT Act), to facilitate the taxation of

‘zero tax companies’ i.e., those companies which show zero or negligible income to avoid

tax. Under MAT, such companies are made liable to pay to the government, by deeming a certain

percentage of their book profit as taxable income. Thus, it is a direct tax.

MAT is applicable to all corporate entities, whether public or private. However, it does not

apply to any income accruing or arising to a company from life insurance business. Nor does it

apply to shipping income liable to tonnage taxation as provided in section 115V to 115VZC of the

IT Act

Page 55: INSTA Revision Plan 3.0 - 2020 INSTA Tests€¦ · • State Agricultural Marketing Board made responsible for grading and standardization. 7. Consider the following statements regarding

www.insightsonindia.com 53 INSTA Revision 3.0

INSIGHTSIAS SIMPLYFYING IAS EXAM PREPARATION

73. Which of the following National Park is largely known for Great Indian Bustard?

(a) Bandipur National Park, Karnataka

(b) Murien National Park, Mizoram

(c) Vedanthangal National Park, Tamil Nadu

(d) Desert National Park, Rajasthan

Solution: D

• Desert National Park, near Jaisalmer and coastal grasslands of the Abdasa and Mandvi

talukas of Kutch District of Gujarat support some populations.

• Great Indian bustard, (Ardeotis nigriceps), large bird of the bustard family (Otididae), one

of the heaviest flying birds in the world. The great Indian bustard inhabits dry grasslands

and scrublands on the Indian subcontinent; its largest populations are found in the Indian

state of Rajasthan.

74. Consider the following statements regarding the Index of Industrial Production (IIP):

1. It is compiled and published every month by Central Statistics Office (CSO).

2. Manufacturing has highest weightage in IIP index.

Which of the statements given above is/are correct?

(a) 1 only

(b) 2 only

(c) Both 1 and 2

(d) Neither 1 nor 2

Solution: C

Index of Industrial Production (IIP) measures the quantum of changes in the industrial

production in an economy and captures the general level of industrial activity in the country.

• The current base year for the IIP series in India is 2011-12. Index of Industrial Production

is compiled and published every month by Central Statistics Office (CSO) of the Ministry

of Statistics and Programme Implementation.

• Industrial production for the purpose of IIP is divided into three sectors, i.e, Mining,

Manufacturing and Electricity. In ‘Sectoral’ classification, relative weights of

Manufacturing, Mining and Electricity are 75.5%, 14.2% and 10.3% respectively.

Page 56: INSTA Revision Plan 3.0 - 2020 INSTA Tests€¦ · • State Agricultural Marketing Board made responsible for grading and standardization. 7. Consider the following statements regarding

www.insightsonindia.com 54 INSTA Revision 3.0

INSIGHTSIAS SIMPLYFYING IAS EXAM PREPARATION

75. ‘Living Planet Report’, sometime seen in the news, is published by which of the following

organisation?

(a) UNEP

(b) Green Watch

(c) IUCN

(d) World Wide Fund for Nature (WWF)

Solution: D

The Living Planet Report, WWF’s flagship publication released every two years, is a

comprehensive study of trends in global biodiversity and the health of the planet. The Living

Planet Report 2018 is the twelfth edition of the report and provides the scientific evidence to

what nature has been telling us repeatedly: unsustainable human activity is pushing the planet’s

natural systems that support life on Earth to the edge.

https://wwf.panda.org/knowledge_hub/all_publications/living_planet_report_2018/

DAY – 34

76. Consider the following statements regarding taxation system

1. Proportional

taxation

: This method has fixed rates for every level of income or production

on which the tax is being imposed

2. Progressive

taxation

: This method has increasing rates of tax for increasing value or

volume on which the tax is being imposed

3. Regressive

taxation

: This method has decreasing rates of tax for increasing value or

volume on which the tax is being imposed

Which of the statements given above is/are correct?

(a) 1 only

(b) 1 and 3 only

(c) 2 and 3 only

(d) 1, 2 and 3

Solution: D

Progressive taxation

• This method has increasing rates of tax for increasing value or volume on which the tax is

being imposed. Indian income tax is a typical example of it. The idea here is less tax on the

people who earn less and higher tax on the people who earn more—classifying income earners

into different slabs.

Page 57: INSTA Revision Plan 3.0 - 2020 INSTA Tests€¦ · • State Agricultural Marketing Board made responsible for grading and standardization. 7. Consider the following statements regarding

www.insightsonindia.com 55 INSTA Revision 3.0

INSIGHTSIAS SIMPLYFYING IAS EXAM PREPARATION

Regressive taxation

• This is just opposite to the progressive method having decreasing rates of tax for increasing

value or volume on which the tax is being imposed

Proportional taxation

• In such a taxation method, there is neither progression nor regression from the point of view

rate of taxes point of view. Such taxes have fixed rates for every level of income or production,

they are neutral from the poor or rich point view or from the point of view of the levels of

production.

77. Which of the following State taxes have been subsumed within the GST?

1. Luxury Tax

2. Purchase Tax

3. Taxes on lotteries, betting and gambling

4. State cesses and surcharges

Select the correct answer using the code given below:

(a) 1, 2 and 3 only

(b) 2, 3 and 4 only

(c) 1, 2 and 4 only

(d) 1, 2, 3 and 4

Solution: D

State taxes that would be subsumed within the GST are:

(a) State VAT

(b) Central Sates Tax

(c) Purchase Tax

(d) Luxury Tax

(e) Entry Tax (All forms)

(f) Entertainment Tax and Amusement Tax (except those levied by the local bodies)

(g) Taxes on advertisements

(h) Taxes on lotteries, betting and gambling

(i) State cesses and surcharges in so far as they relate to supply of goods and services.

Page 58: INSTA Revision Plan 3.0 - 2020 INSTA Tests€¦ · • State Agricultural Marketing Board made responsible for grading and standardization. 7. Consider the following statements regarding

www.insightsonindia.com 56 INSTA Revision 3.0

INSIGHTSIAS SIMPLYFYING IAS EXAM PREPARATION

78. Which of the following passes is/are located in Ladakh?

1. Khardung La

2. Bara-Lacha-La

3. Zoji – La

Which of the statements given above is/are correct?

(a) 1 only

(b) 1, 2 and 3

(c) 1 and 2 only

(d) 2 only

Solution: B

79. Consider the following statements regarding Vivad Se Vishwas scheme

1. The scheme aims to settle the huge number of pending indirect tax cases.

2. It offers a complete waiver on interest and penalty to the taxpayers who pay their

pending taxes within specified time.

Which of the statements given above is/are correct?

Page 59: INSTA Revision Plan 3.0 - 2020 INSTA Tests€¦ · • State Agricultural Marketing Board made responsible for grading and standardization. 7. Consider the following statements regarding

www.insightsonindia.com 57 INSTA Revision 3.0

INSIGHTSIAS SIMPLYFYING IAS EXAM PREPARATION

(a) 1 only

(b) 2 only

(c) Both 1 and 2

(d) Neither 1 nor 2

Solution: B

Vivad Se Vishwas scheme

• The vivad se vishwas scheme was announced by Union Finance Minister Nirmala Sitharaman

during her budget speech on February 1, 2020. The scheme aims to settle the huge number of

pending direct tax cases.

About Vivad Se Vishwas Scheme: The Direct Tax Vivad Se Vishwas Bill, 2020:

• The amnesty scheme, at present, covers disputes pending at the level of commissioner

(appeals), Income Tax Appellate Tribunals (ITAT), high courts, the Supreme Court

and those in international arbitration.

• It offers a complete waiver on interest and penalty to the taxpayers who pay their

pending taxes by March 31.

• The scheme aims to benefit those whose tax demands are locked in dispute in multiple forums.

• If a taxpayer is not able to pay direct taxes by March 31st then, he will get further time till

June 30th. However, in that case, he would have to pay 10 percent more on the tax.

How much?

• In case it is just the interest and the penalty which is in dispute, the taxpayer will have to pay

25% of the disputed amount till March 31, and subsequently, it will be 30%.

• If a taxpayer is not able to pay within the March 31 deadline, he gets a further time till June

30, but in that case, he would have to pay 10% more on the tax.

• In case it is just the interest and the penalty which is in dispute, the taxpayer will have to pay

25% of the disputed amount till March 31, and subsequently, it will be 30%.

Significance:

• The scheme aims to resolve 483,000 direct tax-related disputes pending in various appellate

forums.

80. Consider the following pairs

Navigation System Developed/Owned by 1. GALILEO : USA

2. IRNSS : India

3. QUASI-ZENITH : China

Which of the pairs given above is/are matched correctly?

(a) 2 only

Page 60: INSTA Revision Plan 3.0 - 2020 INSTA Tests€¦ · • State Agricultural Marketing Board made responsible for grading and standardization. 7. Consider the following statements regarding

www.insightsonindia.com 58 INSTA Revision 3.0

INSIGHTSIAS SIMPLYFYING IAS EXAM PREPARATION

(b) 1 only

(c) 1 and 2 only

(d) None

Solution: A

GLONASS is an acronym, which stands for Globalnaya Navigazionnaya Sputnikovaya Sistema,

or Global Navigation Satellite System. GLONASS is Russia’s version of GPS (Global Positioning

System).

List of Global Navigation Satellite Systems:

• GPS of the United States of America.

• Galileo of the European Union.

• IRNSS or NAVIC of India.

• Quasi-Zenith Satellite System (QZSS) of Japan

81. Which of the following forms the part of revenue expenditure?

1. Interest payment by the government.

2. Subsidies forwarded by the government.

3. Law and order expenditures.

4. Grants given by the government.

Select the correct answer using the code given below:

(a) 1, 2 and 3 only

(b) 2, 3 and 4 only

(c) 1, 2 and 4 only

(d) 1, 2, 3 and 4

Solution: D

Revenue expenditure

All expenditures incurred by the government are either of revenue kind or current kind or

compulsive kind. The basic identity of such expenditures is that they are of consumptive kind and

do not involve creation of productive assets. They are either used in running of a productive process

or running a government. A broad category of things that fall under such expenditures in India

are:

Page 61: INSTA Revision Plan 3.0 - 2020 INSTA Tests€¦ · • State Agricultural Marketing Board made responsible for grading and standardization. 7. Consider the following statements regarding

www.insightsonindia.com 59 INSTA Revision 3.0

INSIGHTSIAS SIMPLYFYING IAS EXAM PREPARATION

(i) Interest payment by the government on the internal and external loans;

(ii) Salaries, Pension and Provident Fund paid by the government to government employees;

(iii) Subsidies forwarded to all sectors by the government;

(iv) Defence expenditures by the government;

(v) Postal Deficits of the government;

(vi) Law and order expenditures (i.e., police & paramilitary);

(vii) Expenditures on social services (includes all social sector expenditures as education,

health care, social security, poverty alleviation, etc.) and general services (tax collection,

etc.);

(viii) Grants given by the government to Indian states and foreign countries.

82. Which of the following forms the part of capital receipts?

1. Loan Recovery

2. Borrowings by the Government

3. Small saving schemes

Which of the statements given above is/are correct?

(a) 1 and 2 only

(b) 1 and 3 only

(c) 2 and 3 only

(d) 1, 2 and 3

Solution: D

The capital receipts in India include the following capital kind of accruals to the government:

(i) Loan Recovery

(ii) Borrowings by the Government

(iii) Other Receipts by the Government

This includes many long-term capital accruals to the government through the Provident Fund (PF),

Postal Deposits, various small saving schemes (SSSs) and the government bonds sold to the public

(as Indira Vikas Patra, Kisan Vikas Patra, Market Stabilisation Bond, etc.).

Page 62: INSTA Revision Plan 3.0 - 2020 INSTA Tests€¦ · • State Agricultural Marketing Board made responsible for grading and standardization. 7. Consider the following statements regarding

www.insightsonindia.com 60 INSTA Revision 3.0

INSIGHTSIAS SIMPLYFYING IAS EXAM PREPARATION

83. Which of the following wildlife sanctuaries and National Parks are located in Assam?

1. Nameri National Park

2. Deepor Beel Bird Sanctuary

3. Gorumara National Park

Select the correct answer using the code given below

(a) 1 and 2 only

(b) 1 only

(c) 2 and 3 only

(d) 1, 2 and 3

Solution: A

Nameri National Park, Manas National Park, Kaziranga National Park, Orang National Park and

Deepor Beel Bird Sanctuary are located in Assam.

Gorumara National Park is located in West Bengal.

https://www.thehindu.com/sci-tech/energy-and-environment/assam-floods-why-we-need-to-act-

fast-to-save-kaziranga-and-its-wildlife/article28720848.ece

84. The term Fiscal Deficit indicates

(a) The difference between the total income of the government (total taxes and non-debt

capital receipts) and its total expenditure.

(b) It indicates the borrowing requirements of the government, excluding interest.

(c) The deficit arises when the government’s revenue expenditure exceeds the total revenue

receipts.

(d) None of the Above

Solution: A

• Fiscal Deficit is the difference between the total income of the government (total taxes and

non-debt capital receipts) and its total expenditure. A fiscal deficit situation occurs when the

government’s expenditure exceeds its income. This difference is calculated both in absolute

terms and also as a percentage of the Gross Domestic Product (GDP) of the country. A

recurring high fiscal deficit means that the government has been spending beyond its means.

Fiscal Deficit formula: How is Fiscal Deficit calculated?

• Fiscal Deficit = Total expenditure of the government (capital and revenue expenditure) – Total

income of the government (Revenue receipts + recovery of loans + other receipts)

Page 63: INSTA Revision Plan 3.0 - 2020 INSTA Tests€¦ · • State Agricultural Marketing Board made responsible for grading and standardization. 7. Consider the following statements regarding

www.insightsonindia.com 61 INSTA Revision 3.0

INSIGHTSIAS SIMPLYFYING IAS EXAM PREPARATION

85. Consider the following statements regarding Guru Gobind Singh

1. Guru Gobind Singh was the 10th Sikh guru.

2. He founded the principles of Khalsa or the Five ‘K’s.

3. He is known for the introduction of the turban to cover hair.

Which of the statements given above is/are correct?

(a) 1 and 2 only

(b) 2 and 3 only

(c) 1 and 3 only

(d) 1, 2 and 3

Solution: D

Guru Gobind Singh

• Guru Gobind Singh was the 10th Sikh guru. He was born at Patna, Bihar, India, on December

22, 1666. His birthday sometimes falls either in December or January or even both months in

the Gregorian calendar. The annual celebration of the Guru’s birthday is based on the

Nanakshahi calendar.

• He became the Sikh guru at the age of nine, following the demise of father, Guru Tegh

Bahadur, the ninth Sikh Guru. He is known for his significant contributions to the Sikh

religion, including the introduction of the turban to cover hair.

• He also founded the principles of Khalsa or the Five ‘K’s. They are: Kesh (uncut hair),

Kangha (a wooden comb for the hair), Kara (an iron bracelet), Kachera (100% cotton tieable

undergarment) (not an elastic one) and Kirpan (an iron dagger large enough to defend oneself).

• He is also responsible to establish the highest order in the Sikh community. Followers of

the Sikh faith religiously follow the morals and codes of discipline set up by Guru Gobind

Singh.

• He fought against the Mughals later in battle of Muktsar in 1705. He was assassinated in

1708.

• He named Guru Granth Sahib, the religious text of the Khalsas and the Sikhs, as the

next Guru of the two communities.

86. Consider the following statements regarding Ways and Means Advances (WMA)

1. It is a facility only for union government to borrow from the RBI.

2. These borrowings are meant to help them to tide over temporary mismatches in cash

flows of their receipts and expenditures.

3. The interest rate on WMA is the RBI’s repo rate.

Which of the statements given above is/are correct?

(a) 1 and 2 only

Page 64: INSTA Revision Plan 3.0 - 2020 INSTA Tests€¦ · • State Agricultural Marketing Board made responsible for grading and standardization. 7. Consider the following statements regarding

www.insightsonindia.com 62 INSTA Revision 3.0

INSIGHTSIAS SIMPLYFYING IAS EXAM PREPARATION

(b) 1 and 3 only

(c) 2 and 3 only

(d) 1, 2 and 3

Solution: C

What exactly is Ways and Means Advances (WMA)?

Simply put, it is a facility for both the Centre and states to borrow from the RBI. These

borrowings are meant purely to help them to tide over temporary mismatches in cash flows of their

receipts and expenditures. In that sense, they aren’t a source of finance per se. Section 17(5) of

the RBI Act, 1934 authorizes the central bank to lend to the Centre and state

governments subject to their being repayable “not later than three months from the date of the

making of the advance”.

How much does the RBI charge on these advances?

The interest rate on WMA is the RBI’s repo rate, which is basically the rate at which it lends

short-term money to banks. That rate is currently 4.4%. The governments are, however, allowed

to draw amounts in excess of their WMA limits. The interest on such overdraft is 2 percentage

points above the repo rate, which now works out to 6.4%. Further, no state can run an overdraft

with the RBI for more than a certain period.

87. The FRBM Act made it mandatory for the government to place which of the following

documents in Parliament annually?

1. Medium Term Fiscal Policy Statement

2. Macroeconomic Framework Statement

3. Fiscal Policy Strategy Statement

Which of the statements given above is/are correct?

(a) 1 and 2 only

(b) 1 and 3 only

(c) 2 and 3 only

(d) 1, 2 and 3

Solution: D

• The Fiscal Responsibility and Budget Management Act (FRBM Act), 2003, establishes

financial discipline to reduce fiscal deficit.

When was the FRBM Act enacted? Who introduced it in India?

• The FRBM Bill was introduced by the then finance minister, Yashwant Sinha, in 2000. The

Bill, approved by the Union Cabinet in 2003, became effective from July 5, 2004.

Page 65: INSTA Revision Plan 3.0 - 2020 INSTA Tests€¦ · • State Agricultural Marketing Board made responsible for grading and standardization. 7. Consider the following statements regarding

www.insightsonindia.com 63 INSTA Revision 3.0

INSIGHTSIAS SIMPLYFYING IAS EXAM PREPARATION

What are the objectives of the FRBM Act?

• The FRBM Act aims to introduce transparency in India’s fiscal management systems. The

Act’s long-term objective is for India to achieve fiscal stability and to give the Reserve Bank

of India (RBI) flexibility to deal with inflation in India. The FRBM Act was enacted to

introduce more equitable distribution of India’s debt over the years.

Key features of the FRBM Act

The FRBM Act made it mandatory for the government to place the following along with the Union

Budget documents in Parliament annually:

1. Medium Term Fiscal Policy Statement

2. Macroeconomic Framework Statement

3. Fiscal Policy Strategy Statement

The FRBM Act proposed that revenue deficit, fiscal deficit, tax revenue and the total outstanding

liabilities be projected as a percentage of gross domestic product (GDP) in the medium-term fiscal

policy statement.

88. Which of the following agency/body finally approves the disinvestment in India?

(a) NITI Ayog

(b) Ministry of Finance

(c) Ministry of Heavy Industries

(d) Cabinet Committee of Economic Affairs (CCEA)

Solution: D

Strategic disinvestment is the sale of a substantial portion of the Government shareholding of

a central public sector enterprise (CPSE) of 50% or more along with transfer of management

control.

• If the government is selling minority shares in a PSE (less than 50%), it will continue to be

the owner of the PSE. This is normal disinvestment procedure.

• The Department of Investment and Public Asset Management (DIPAM) under the Ministry

of Finance has been made the nodal department for the strategic stake sale in the Public

Sector Undertakings (PSUs).

• DIPAM and NITI Ayog will jointly identify PSUs for strategic sale.

• Cabinet Committee of Economic Affairs (CCEA) is mandated to approve strategic

disinvestment of CPSEs.

89. Consider the following statements regarding Public Debt Management

1. As per Reserve Bank of India Act of 1934, the Reserve Bank is the public debt manager

for the Union government.

Page 66: INSTA Revision Plan 3.0 - 2020 INSTA Tests€¦ · • State Agricultural Marketing Board made responsible for grading and standardization. 7. Consider the following statements regarding

www.insightsonindia.com 64 INSTA Revision 3.0

INSIGHTSIAS SIMPLYFYING IAS EXAM PREPARATION

2. The Union government’s liabilities account for a little over 26% of the country’s GDP.

Which of the statements given above is/are correct?

(a) 1 only

(b) 2 only

(c) Both 1 and 2

(d) Neither 1 nor 2

Solution: A

Importance of Public Debt Management in India

• As per Reserve Bank of India Act of 1934, the Reserve Bank is both the banker and public

debt manager for the Union government. The RBI handles all the money, remittances, foreign

exchange and banking transactions on behalf of the Government. The Union government also

deposits its cash balance with the RBI. However, of late, there is a demand for creating a

specialized agency for managing public debt as exists in some advanced economies. For

instance, the Niti Aayog has advocated the creation of a separate public debt management

agency (PDMA).

Public Debt versus Private Debt

• Public Debt is the money owed by the Union government, while private debt comprises of all

the loans raised by private companies, corporate sector and individuals such as home loans,

auto loans, personal loans.

Public Debt as a percentage of GDP

• The Union government’s liabilities account for a little over 46% of the country’s GDP.

However, if the public debt is calculated as general government liabilities, which also includes

the liabilities of states then it goes up to 68% of the country’s GDP.

90. Consider the following statements regarding National Data and Analytics Platform (NDAP)

1. It has been launched by Ministry of Personnel, Public Grievances

2. It aims to democratize access to publicly available government data

Which of the statements given above is/are correct?

(a) 1 only

(b) 2 only

(c) Both 1 and 2

(d) Neither 1 nor 2

Solution: B

Page 67: INSTA Revision Plan 3.0 - 2020 INSTA Tests€¦ · • State Agricultural Marketing Board made responsible for grading and standardization. 7. Consider the following statements regarding

www.insightsonindia.com 65 INSTA Revision 3.0

INSIGHTSIAS SIMPLYFYING IAS EXAM PREPARATION

National Data and Analytics Platform:

Recently, the NITI Aayog released the National Data and Analytical Platform (NDAP) vision

document.

• It aims to democratize access to publicly available government data.

• It will host the latest datasets from various government websites, present them

• Coherently and provide tools for analytics and visualization.

• It will follow a user-centric approach and will enable data access in a simple and intuitive

portal tailored to the needs of a variety of stakeholders.

• It will spearhead the standardization of formats in which data is presented across sectors

• It would cater to a wide audience of policy makers, researchers, innovators, data scientists,

journalists and citizens.

91. Consider the following statements regarding Double Taxation Avoidance Agreement (DTAA)

1. The objective of DTAA is that tax-payers in these countries can avoid being taxed twice

for the same income.

2. DTAAs are intended to make a country an attractive investment destination by

providing relief on dual taxation.

Which of the statements given above is/are correct?

(a) 1 only

(b) 2 only

(c) Both 1 and 2

(d) Neither 1 nor 2

Solution: C

• India recently amended its Double Taxation Avoidance Agreement (DTAA) with

Mauritius to plug certain loopholes.

What is it?

• A DTAA is a tax treaty signed between two or more countries. Its key objective is that tax-

payers in these countries can avoid being taxed twice for the same income. A DTAA applies in

cases where a tax-payer resides in one country and earns income in another.

• DTAAs can either be comprehensive to cover all sources of income or be limited to certain

areas such as taxing of income from shipping, air transport, inheritance, etc. India has DTAAs

with more than eighty countries, of which comprehensive agreements include those with

Australia, Canada, Germany, Mauritius, Singapore, UAE, the UK and US.

Why is it important?

• DTAAs are intended to make a country an attractive investment destination by providing

relief on dual taxation. Such relief is provided by exempting income earned abroad from tax

Page 68: INSTA Revision Plan 3.0 - 2020 INSTA Tests€¦ · • State Agricultural Marketing Board made responsible for grading and standardization. 7. Consider the following statements regarding

www.insightsonindia.com 66 INSTA Revision 3.0

INSIGHTSIAS SIMPLYFYING IAS EXAM PREPARATION

in the resident country or providing credit to the extent taxes have already been paid abroad.

DTAAs also provide for concessional rates of tax in some cases.

• For instance, interest on NRI bank deposits attract 30 per cent TDS (tax deduction at source)

here. But under the DTAAs that India has signed with several countries, tax is deducted at

only 10 to 15 per cent. Many of India’s DTAAs also have lower tax rates for royalty, fee for

technical services, etc.

• Favorable tax treatment for capital gains under certain DTAAs such the one with

Mauritius have encouraged a lot of foreign investment into India. Mauritius accounted for

$93.65 billion or one-third of the total FDI flows into India between April 2000 and December

2015. It has also remained a favored route for foreign portfolio investors. But the problem is

DTAAs can become an incentive for even legitimate investors to route investments through

low-tax regimes to sidestep taxation. This leads to loss of tax revenue for the country.

92. Which of the following is/are part of the Personal Disposable Income?

1. Transfer payments to the households from the government and firms

2. Corporate Tax.

3. Personal Tax Payments.

4. Net Interest payments made by households

Select the correct answer using the code given below.

(a) 2 and 4 only

(b) 1 and 3 only

(c) 2 only

(d) 1 only

Solution: D

Personal Income (PI) ≡ NI – Undistributed profits – Net interest payments made by households

– Corporate tax + Transfer payments to the households from the government and firms.

• If we deduct the Personal Tax Payments (income tax, for example) and Non-tax Payments

(such as fines) from PI, we obtain what is known as the Personal Disposable Income.

Personal Disposable Income (PDI ) ≡ PI – Personal tax payments – Non-tax payments.

93. Tritiya Ratna, Shetkarayacha Aasud and Cultivator’s Whipcord these books are written by

(a) Bankim Chandra Chatterjee

(b) Jyotirao Phule

(c) Bal Gangadhar Tilak

(d) Dayananda Saraswati

Page 69: INSTA Revision Plan 3.0 - 2020 INSTA Tests€¦ · • State Agricultural Marketing Board made responsible for grading and standardization. 7. Consider the following statements regarding

www.insightsonindia.com 67 INSTA Revision 3.0

INSIGHTSIAS SIMPLYFYING IAS EXAM PREPARATION

Solution: B

Jyotirao Phule:

• Born in 1827 in Satara district of Maharashtra. Phule was given the title of Mahatma on

May 11, 1888, by Vithalrao Krishnaji Vandekar, a Maharashtrian social activist.

Social reforms and key contributions:

• His work is related mainly to eradication of untouchability and caste system,

emancipation and empowerment of women, reform of Hindu family life.

• Along with his wife, Savitribai Phule, he is regarded as pioneers of women’s education in

India.

• The couples were the first native Indians to open the first indigenously-run school for girls in

India in August 1848 at Pune in Maharashtra.

• Later, the Phules started schools for children from the then untouchable castes such as Mahar

and Mang.

• In 1863, he opened a home for pregnant Brahmin widows to give birth in a safe and secure

place.

• He opened an orphanage home to avoid infanticide. In this regard, he is believed to be the first

Hindu to start an orphanage for the unfortunate children.

• In 1868, Jyotirao decided to construct a common bathing tank outside his house to exhibit his

embracing attitude towards all human beings and wished to dine with everyone, regardless of

their caste.

• In 1873, Phule founded the Satyashodhak Samaj, or the Society of Seekers of Truth, for

the rights of depressed classes, to denounce the caste system and to spread rational thinking.

His famous works:

• Tritiya Ratna (1855), Gulamgiri (1873), Shetkarayacha Aasud, or Cultivator’s Whipcord

(1881), Satyashodhak Samajokt Mangalashtakasah Sarva Puja-vidhi (1887).

94. Which of the following measures can be adopted to promote the Fiscal Consolidation?

1. Stopping the leakages in Subsidies

2. Agriculture Loan Waiver

3. Increasing the tax base

Select the correct answer using the codes given below.

(a) 2 and 3 only

(b) 1 and 3 only

(c) 2 only

(d) 1, 2 and 3

Page 70: INSTA Revision Plan 3.0 - 2020 INSTA Tests€¦ · • State Agricultural Marketing Board made responsible for grading and standardization. 7. Consider the following statements regarding

www.insightsonindia.com 68 INSTA Revision 3.0

INSIGHTSIAS SIMPLYFYING IAS EXAM PREPARATION

Solution: B

Fiscal Consolidation refers to the policies undertaken by Governments to reduce their deficits

and accumulation of debt stock.

• Agriculture Loan Waiver will increase the fiscal burden on the government.

• Stopping the leakages in Subsidies and increasing the tax base will increase the fiscal resource

of the government. Thus, help in fiscal consolidation.

95. Match the following national parks with their states

1. Kanchendzonga National Park : Sikkim

2. Neora Valley National Park : Assam

3. Namdapha National Park : Arunachal Pradesh

4. Singalila National Park : West Bengal

Which of the pairs given above is/are correctly matched?

(a) 1, 2 and 3 only

(b) 2, 3 and 4 only

(c) 1, 3 and 4 only

(d) 1, 2, 3 and 4

Solution: C

Khangchendzonga National Park also Kanchenjunga Biosphere Reserve is a National Park

and a Biosphere reserve located in Sikkim, India. It was inscribed to the UNESCO World

Heritage Sites list in July 2016, becoming the first “Mixed Heritage” site of India.

Neora Valley National Park is situated in the Kalimpong district, West Bengal, India and was

established in 1986. It spreads over an area of 88 km² and is one of the richest biological zones in

the entire Eastern India.

Namdapha National Park is a 1,985 km² large protected area in Arunachal Pradesh of

Northeast India. With more than 1,000 floral and about 1,400 faunal species, it is a biodiversity

hotspot in the Eastern Himalayas. The national park harbours the northernmost lowland

evergreen rainforests in the world at 27°N latitude.

Singalila National Park is a national park of India located on the Singalila Ridge at an altitude

of more than 7000 feet above sea level, in the Darjeeling district of West Bengal. It is well known

for the trekking route to Sandakphu that runs through it.

96. Consider the following statements regarding the Capital Gains Tax:

1. It is levied on both short term and long term gains.

2. Capital gains tax in India need be paid if the person who has inherited the property

decides to sell it.

Page 71: INSTA Revision Plan 3.0 - 2020 INSTA Tests€¦ · • State Agricultural Marketing Board made responsible for grading and standardization. 7. Consider the following statements regarding

www.insightsonindia.com 69 INSTA Revision 3.0

INSIGHTSIAS SIMPLYFYING IAS EXAM PREPARATION

Which of the statements given above is/are correct?

(a) 1 only

(b) 2 only

(c) Both 1 and 2

(d) Neither 1 nor 2

Solution: C

Capital gain can be defined as any profit that is received through the sale of a capital asset.

The profit that is received falls under the income category. Therefore, a tax needs to be paid on

the income that is received. The tax that is paid is called capital gains tax and it can either be long

term or short term.

Under the Income Tax Act, capital gains tax in India need not be paid in case the individual

inherits the property and there is no sale. However, if the person who has inherited the property

decides to sell it, tax will have to be paid on the income that has been generated from the sale.

Some of the examples of capital assets are jewellery, machinery, leasehold rights, trademarks,

patents, vehicles, house property, building, and land.

97. Which of the following category of people are not exempted from income tax?

(a) Income of a member of a Scheduled Tribes who resides in any area in the State of Assam

(b) Income of a member of a Scheduled Tribes who resides in any area in the State of

Madhya Pradesh

(c) Income of a member of a Scheduled Tribes who resides in any area in the State of

Meghalaya

(d) Income of a member of a Scheduled Tribes who resides in any area in the State of

Manipur

Solution: B

Income of a member of a Scheduled Tribe [as per article 366(25) of the Constitution] is

exempt from tax, if following conditions are satisfied:

• Such member resides in any area in the State of Nagaland, Manipur, Tripura, Arunachal

Pradesh, Mizoram or district of North Cachar Hills, Mikir Hills, Khasi Hills, Jaintia Hills and

Garo Hills or in the Ladakh region of the State of Jammu and Kashmir.

• Such exemption is available in respect of income which accrues/arises from any source in such

areas or income by way of dividends/interest on securities arises from any area.

Page 72: INSTA Revision Plan 3.0 - 2020 INSTA Tests€¦ · • State Agricultural Marketing Board made responsible for grading and standardization. 7. Consider the following statements regarding

www.insightsonindia.com 70 INSTA Revision 3.0

INSIGHTSIAS SIMPLYFYING IAS EXAM PREPARATION

98. Istanbul Convention is often seen in the news, is related to

(a) Cyber crime

(b) Crypto currency

(c) Protection of children

(d) violence against women

Solution: D

Istanbul Convention:

• Poland is to withdraw from Istanbul Convention- a treaty aimed at preventing

violence against women.

What’s the issue?

• The reason behind withdrawal is that Poland thinks the Convention is harmful because it

required schools to teach children about gender. Also, it says, the treaty tries to construct a

“socio-cultural gender against the biological gender”. For example, some items of the

convention foresee educating children and young people about forming homosexual families.

What is the Istanbul Convention?

• It is also called as the Council of Europe Convention on preventing and combating

violence against women and domestic violence.

• The treaty is the world’s first binding instrument to prevent and tackle violence

against women.

• It is the most comprehensive legal framework that exists to tackle violence against women

and girls, covering domestic violence, rape, sexual assault, female genital mutilation (FGM),

so-called honour-based violence, and forced marriage.

• The Convention sets minimum standards for governments to meet when tackling violence

against women.

• When a government ratifies the Convention, they are legally bound to follow it.

99. Which of the following constitute Capital Account?

1. Trade in Services

2. Transfer payments

3. Bilateral loans

4. Foreign Direct Investment (FDI)

Select the correct answer using the codes given below:

(a) 3 and 4 only

(b) 1, 2 and 4 only

(c) 2 and 3 only

Page 73: INSTA Revision Plan 3.0 - 2020 INSTA Tests€¦ · • State Agricultural Marketing Board made responsible for grading and standardization. 7. Consider the following statements regarding

www.insightsonindia.com 71 INSTA Revision 3.0

INSIGHTSIAS SIMPLYFYING IAS EXAM PREPARATION

(d) 1, 2, 3 and 4

Solution: A

Capital Account records all international transactions of assets.

Current Account is the record of trade in goods and services and transfer payments.

Source: NCERT Macroeconomics

100. Consider the following statements regarding the SATYABHAMA portal

1. It has been launched by NITI Aayog

2. It allows online submission of project proposals along with monitoring of the projects

and utilization of funds / grants.

Which of the statements given above is/are correct?

(a) 1 only

(b) 2 only

(c) Both 1 and 2

(d) Neither 1 nor 2

Solution: B

Page 74: INSTA Revision Plan 3.0 - 2020 INSTA Tests€¦ · • State Agricultural Marketing Board made responsible for grading and standardization. 7. Consider the following statements regarding

www.insightsonindia.com 72 INSTA Revision 3.0

INSIGHTSIAS SIMPLYFYING IAS EXAM PREPARATION

SATYABHAMA portal: Launched by Ministry of Mines.

• SATYABHAMA stands for Science and Technology Yojana for Aatmanirbhar Bharat in

Mining Advancement.

• Designed, developed and implemented by National Informatics Centre (NIC), Mines

Informatics Division.

• It allows online submission of project proposals along with monitoring of the projects and

utilization of funds / grants. The researchers can also submit progress reports and Final

Technical Reports of the projects in the electronic format in the portal.

Page 75: INSTA Revision Plan 3.0 - 2020 INSTA Tests€¦ · • State Agricultural Marketing Board made responsible for grading and standardization. 7. Consider the following statements regarding

Total Tests - 12 (10 Theme Based + 2 Full Syllabus)

Comprehensive coverage of ALL THEMES based on which 20-30

questions are asked in Preliminary Examination every year.

Mains self-study questions with every mock test to facilitate

integrated preparation for both Prelims and Mains.

Enables students to inter-link factual knowledge, develop deeper

understanding, improve knowledge retention and gain competitive

edge.

INSIGHTSIAS PRELIMS THEMATIC

TEST SERIES (PTS) 2020

INSIGHTSIASSIMPLIFYING IAS EXAM PREPARATION

thEarly Bird Offer : 15 % Discount till 26 June 2020Old Subscriber Offer : 20 % Discount Fee: 3,500/- incl. GST

thStarts from 28 June 2020

7483163074 / 9380863034 [email protected] www.insightsonindia.com

Subscribe HERE instacourses.insightsonindia.com

* PTS is free for all our prelims 2020 and IPM 2020 full package subscribers